h1

The Date of the Exodus (2/2)

February 19, 2015

This article is the second of two in a consideration of the date of the Exodus. Typically, only two dates are considered viable; c.1440 BCE (the ‘early date’), and c.1280 BCE (the ‘late date’). Arguments for these dates are reviewed and compared here.

Summary of Key Arguments

Here are the key arguments for the early date (c. 1440 BCE), together with criticisms.

Argument: The 480 years of 1 Kings 1:6 indicates an early date

  • This is only a relative chronology, and the number does not agree with the years recorded in the Judges[1] [2]
  • The number may be symbolic for 12 generations[3]
  • This disagrees with the date for Abraham[4]

Argument: Some destruction layers in Canaan support an early date[5]

  • Destruction and occupation layers provide more support for a late date[6]
  • No evidence for Edom and Moab existing at an early date[7]

Argument: Reference to the Habiru in the 14th century Armana letters[8]

  • Extensive study has revealed no direct correspondence between the Habiru and the Hebrews, thus the Armana letters do not support the early date[9] [10]
  • Armarna correspondence contradicts early date destruction of Hazor[11]

Argument: No evidence for 13th century occupation or destruction of Jericho, Ai, or Hazor[12]

  • Hazor was occupied and destroyed in the 13th century,[13] reliable excavation of 13th century Jericho is challenged by extensive erosion,[14] and the archaeological data for Ai is difficult to reconcile with both early and late dates;[15] these are insignificant archaeological challenges for the late date

Here are the key arguments for the late date (c. 1280 BCE), together with criticisms.

Argument: Destruction of Hazor in 13th century[16]

  • The archaeological evidence may indicate the destruction in Judges 4:24[17]

Argument: Pithom and Rameses in Exodus 1:11 are evidence for events under Pharaoh Rameses II (1279-1213 BCE)[18] [19]

  • This requires Rameses to be built before Rameses II even began to rule[20]
  • Unlike the pharaoh of the Exodus, Rameses II did not die in the Red Sea[21]
  • The sites were built earlier; one was later renamed ‘Rameses’, and an editor of the book of Exodus updated the text with this name[22]

Argument: Covenant formulas in the Law of Moses closely match those from 1400-1200 BCE[23]

  • There is an insufficient match to date the Biblical covenants precisely[24]

Argument: The Merneptah Stele (13th century), is the earliest reference to Israel in Canaan[25]

  • Egyptologist Manfred Görg has suggested an Egyptian inscription he dates to the 13th century, contains a reference to Israel which may have been copied from an 18th Dynasty record (16th-13th centuries BCE), implying Israel was in Canaan before the 13th century[26]

Argument: Egypt occupied Canaan until the 12th century[27]

  • This remains unaddressed by key proponents of the early date;[28] this contradicts completely a late date for the Exodus

Argument: A very large population entered Canaan in the late 13th century[29]

  • This remains unaddressed by key proponents of the early date; [30] there is no evidence for such a population entering Canaan in the 15th century

Review of Arguments

The early date is highly vulnerable to a range of criticisms, and has the least archaeological support. In particular, the occupation and control of Canaan by Egypt until the end of the 13th century, the lack of any evidence for a new population entering Canaan in the 15th century, the Armana correspondence, the non-existence of Edom and Moab in the 15th century, and the evidence for destruction of Canaanite sites matching a 13th century conquest rather than a 15th century conquest, are  formidable challenges to the traditional late date.

Objects to the late date are less substantial. There is no evidence that Rameses as a place name in Exodus 1 is a later editorial gloss.[31] Görg’s suggestion of an Egyptian reference to Israel earlier than the Merneptah Stele is problematic. [32] [33] Wood’s attribution of the 13th century Hazor destruction level to Deborah and Barak fails to provide evidence.[34] [35] His dating of Pithom and Rameses on the basis of the birth of Moses being described later in Exodus 1, assumes an unnecessarily strict chronological sequence for the narrative.

The pharaoh under whom Pithom and Rameses were built died while Moses was in the wilderness before the Exodus[36] (matching Rameses II and a late date Exodus). Additionally, Hoffmeier argues Exodus does not represent the pharaoh of the Exodus as dying in the Red Sea,[37] whereas an early date pharaoh would have to be Thutmose III or Amenhotep II, neither of whom died by drowning.[38]

Conclusion

Although vigorous debate over date of the Exodus is ongoing, the 13th century date continues to be held widely among those scholars who accept the historicity of the Exodus.[39] As early as 1999 Hoffmeier observed ‘Dating the period of the oppression and exodus to the fifteenth century B.C. has largely been replaced in favor of a thirteenth-century date’.[40]


[1] The years of the judges, if added sequentially, result in 633-650 years between the exodus and the reign of Solomon, compelling supporters of the early date to read the years in Judges less literally, in order to read the years in 1 Kings 6:1 more literally; ‘To get around the dilemma caused by the difference between 480 and 633-650 years, advocates of the 15th-century (and the later date) exodus date are forced to harmonize the conflicting data by proposing some overlap between judgeships to bring the 480-year figure into alignment with the 633–650 year total.12  By doing this, one abandons a straightforward, literal reading of the Judges through Exodus narratives.’, Hoffmeier, ‘What is the Biblical Date For the Exodus? A Response to Bryant Wood’, Journal of the Evangelical Theological Society, 50/2 (2007), 228.

[2] ‘When one seeks to reconstruct the numbers given in the biblical accounts, consistently and literally, they do not add up to the number 480 given in 1 Kgs 6:1.’, Hawkins, ‘Propositions For Evangelical Acceptance Of A Late-Date Exodus-Conquest:  Biblical Data And The Royal Scarabs From Mt. Ebal’, Journal of the Evangelical Theological Society, 50/1 (2007), 35.

[3] ‘It has long been thought that the 480-year figure of 1 Kgs 6:1 might be a symbolic figure that derives from 12 times 40-40 years being a symbolic number for a generation—thus signifying that 12 generations had elapsed between the exodus and Solomon’s 4th year. Since men were usually married and had children by age 20–25, 60  a period closer to 300 years would be more accurate. When one adds 300 to 967 BC, an Exodus date around 1267 BC (20 years into the reign of Ramesses II) results.’, ibid., p. 236.

[4] ‘A 15th-century B.C. date presents problems for the chronology of Abram. Archaeological evidence relating to the overthrow of Sodom and Gomorrah seems to date Abram’s arrival in Canaan around 1900 B.C. The Genesis narratives place Jacob’s migration to Egypt about 215 years later. On the basis of the 430 years of Exodus 12:40 it would seem that Abram came to Canaan about 2086 B.C., some 645 years before the exodus. That would date his birth (cf. 12:4) about 2161 B.C. If the Sodom and Gomorrah evidence is correct, Abram’s arrival in Canaan would harmonize with a 13th-century B.C. date.’, Harrison, ‘Exodus, The’, in Elwell & Beitzel, Baker Encyclopedia of the Bible (1988), 743-744.

[5]According to Wood, some archaeological findings—such as destruction layers from Jericho, Ai and Hazor—support a 15th-century exodus (Wood, “The Rise and Fall of the 13th-Century Exodus-Conquest Theory,” 488–89; Wood, “From Ramesses to Shiloh,” 256–82).’, Thornhill, ‘Exodus’, in Barry & Wentz (eds.), The Lexham Bible Dictionary (2012).

[6] ‘The destruction and occupation layers of many conquest cities (e.g., Lachish, Debir, Hazor, Bethel, etc.) favor the 13th-century dating.’, ibid.

[7] ‘Excavation findings seem to indicate that Edom and Moab (compare Exod 13:15; Num 20:14–21) were not yet established peoples during the mid-14th century.’, ibid.

[8] ‘Wood also cites the mention of the ‘ (‘)apiru in the Canaanite Amarna letters of the mid-14th century, as well as an inscription dating to the 18th Dynasty. This inscription appears to mention Ashkelon, Canaan, and Israel (Wood, “The Rise and Fall of the 13th-Century Exodus-Conquest Theory,” 489).’, Thornhill, ‘Exodus’, in Barry & Wentz (eds.), The Lexham Bible Dictionary (2012).

[9] ‘The relationship between the “Habiru” of the Amarna letters, the “Apiru” in 13th-century B.C. Egypt, and the biblical Hebrews has been examined minutely by scholars. Widely differing opinions have been offered. Some believe that the three are variations of the name of one people. To others, however, it seems far from clear that there was any significant relationship between the names. Such disagreement also tends to intensify the problem.’, Harrison, ‘Exodus, The’, in Elwell & Beitzel, Baker Encyclopedia of the Bible (1988), 744.

[10] ‘The ʿapiru (sometimes ḫapiru or ḫabiru) are considered to be warlords, brigands and disenfranchised peoples on the outskirts of society. Rainey has demonstrated that the term cannot be etymologically related to “Hebrew,” and the range of use of the term makes it clear that the ʿapiru cannot be equated with Israelites. Nevertheless, some would contend that it does not entirely rule out the possibility that Israelites, along with other peoples, could have been designated by the term.’, Walton, ‘Exodus, Date of’, in Alexander & Baker, Dictionary of the Old Testament: Pentateuch (2003), 263.

[11]According to the Wood, the marauding Habiru of the Amarna Letters could be the Hebrews.113  Abi-Milku, however, makes clear that Hazor was an ally of the Habiru rather than being the destroyers of Hazor. This information from the Amarna correspondences demonstrates that Hazor during the LB IIA was a major player in the region and does not sound like a city that had just been demolished and burnt by Joshua and his forces.’, Hoffmeier, ‘What is the Biblical Date For the Exodus? A Response to Bryant Wood’, Journal of the Evangelical Theological Society, 50/2 (2007), 245.

[12] ‘Only three cities are recorded as having been destroyed by fire by the Israelites: Jericho (Josh 6:24); Ai (Josh 8:28); and Hazor (Josh 11:11).14  All three pose problems for a late 13th-century conquest. At Jericho and Ai, no evidence has been found for occupation in the late 13th century, let alone for a destruction at that time.’, Wood, ‘The Rise and Fall of the 13th-Century Exodus-Conquest Theory’, Journal of the Evangelical Theological Society, 48/3 (2005), 477.

[13] ‘In Canaan, the drastic destruction of Hazor (level 13) in the later 13th century B.C. (despite misconceptions to the contrary) may well reflect Joshua’s exploit.’,  Kitchen, ‘Exodus, The,’, in Freedman (ed.), Anchor Yale Bible Dictionary (1992), 702.

[14] ‘at Jericho, nearly half a millennium of erosion has long since removed virtually all pertinent evidence.’, ibid., p. 702.

[15] ‘Ai remains an enigma on any view;’, ibid., p. 702.

[16] ‘In Canaan, the drastic destruction of Hazor (level 13) in the later 13th century B.C. (despite misconceptions to the contrary) may well reflect Joshua’s exploit.’,  Kitchen, ‘Exodus, The,’, in Freedman (ed.), Anchor Yale Bible Dictionary (1992), 702.

[17]Following the 1230 bc destruction, there was no urban center there until the time of Solomon in the 10th century bc (1 Kgs 9:15).16  The defeat of Jabin, king of Hazor, by a coalition of Hebrew tribes under the leadership of Deborah and Barak is recorded in Judges 4–5. Judges 4:24 indicates that the Israelites destroyed Hazor at this time: “And the hand of the Israelites grew stronger and stronger against Jabin, the Canaanite king, until they destroyed him.”17  If Joshua destroyed Hazor in 1230 bc, then there would be no city for the Jabin of Judges 4 to rule.’, Wood, ‘The Rise and Fall of the 13th-Century Exodus-Conquest Theory’, Journal of the Evangelical Theological Society, 48/3 (2005), 477.

[18] ‘Egyptologists have long understood the reference to Rameses to refer to Pi-Ramesses, the delta metropolis built by Ramesses II, the 19th Dynasty monarch who reigned from 1279–1213 BC.’, Hoffmeier, ‘What is the Biblical Date For the Exodus? A Response to Bryant Wood’, Journal of the Evangelical Theological Society, 50/2 (2007), 231.

[19] ‘The archaeological data is now unequivocal: Pi-Ramesses is located at modern-day Qantir, near Faqus, and was built by Ramesses II beginning around 1270 BC;’, ibid., pp. 232-233.

[20] ‘Since Moses was 80 years of age at the time of the exodus (Exod 7:7), the building of Rameses would have taken place well before Moses’ birth in 1340 bc (according to the 13th-century theory), long before Rameses came to the throne.’, Wood, ‘The Rise and Fall of the 13th-Century Exodus-Conquest Theory’, Journal of the Evangelical Theological Society, 48/3 (2005), 478.

[21] ‘Obviously, Rameses II did not drown in the yam sup, [commonly translated ‘Red Sea’] as he died of natural causes some 47 years after the presumed exodus date of 1260 bc.’, ibid., p. 478.

[22] ‘It is clear, then, that the name Rameses used in Exod 1:11 is an editorial updating of an earlier name that went out of use.’, ibid., p. 478.

[23] ‘Scholars have understood for some time, since the work of Mendenhall, Kline, and Kitchen, that the book of Deuteronomy has the literary and legal form that characterized late second millennium BC Hittite international treaties.’,  Niehaus, ‘Covenant and Narrative, God and Time’, Journal of the Evangelical Theological Society 53/3 (2010), 550.

[24] ‘The format of the biblical material is varied and complex and cannot be dated to a particular time period based on ANE treaty documents’, Wood, ‘The Rise and Fall of the 13th-Century Exodus-Conquest Theory’, Journal of the Evangelical Theological Society, 48/3 (2005), 480-481.

[25] ‘The Merneptah stela is also cited as evidence for this date, since Israel is referenced as a people group rather than a nation.’, Thornhill, ‘Exodus’, in Barry & Wentz (eds.), The Lexham Bible Dictionary (2012).

[26] ‘Due to the similarity of these names to the names on the Merenptah stela, Gorg suggests the name list may derive from the time of Rameses II, but adopting an older name sequence from the 18th Dynasty. This evidence, if it holds up to further scrutiny, would also support a 15th-century bc exodus-conquest rather than a 13th-century bc timeframe.’, Wood, ‘The Rise and Fall of the 13th-Century Exodus-Conquest Theory’, Journal of the Evangelical Theological Society, 48/3 (2005), 489.

[27] ‘In trying to work out an evangelical understanding of the emergence of Israel, Mark Chavalas and Murray Adamthwaite have recently noted that certain conditions in the archaeology of Palestine appear to mitigate against the traditional early date positioning of the Exodus/Conquest.12  They note that, at a series of sites all over Palestine, “the clear picture is that Egyptian occupation continued until the end of the Late Bronze Age (1200 BC).”’, Hawkins, ‘Propositions For Evangelical Acceptance Of A Late-Date Exodus-Conquest:  Biblical Data And The Royal Scarabs From Mt. Ebal’, Journal of the Evangelical Theological Society, 50/1 (2007), 34.

[28] For example, it is never mentioned by Bryant Wood (foremost proponent of the early date), in his key articles ‘The Rise and Fall of the 13th-Century Exodus-Conquest Theory’, Journal of the Evangelical Theological Society, 48/3 (2005), 475-489, and ‘The Biblical Date For The Exodus Is 1446 BC:  A Response To James Hoffmeier’, Journal of the Evangelical Theological Society, 50/2 (2005), 249-258.

[29] ‘The implication seemed clear that a new population group had arrived in the Central Hill-Country during the transition from the Late Bronze Age to the Iron Age I.’, Hawkins, ‘Propositions For Evangelical Acceptance Of A Late-Date Exodus-Conquest:  Biblical Data And The Royal Scarabs From Mt. Ebal’, Journal of the Evangelical Theological Society, 50/1 (2007), 34.

[30] ‘While this material has seemed to point toward a late date for Israel’s emergence in Canaan,9  it has largely gone unnoticed by evangelical scholars writing histories of Israel10  or commentaries on Joshua.’, ibid., p. 34.

[31] ‘The toponym Rameses (רַעַמְס) occurs five times in the OT, in Gen 47:11; 53  Exod 1:11; 12:37; and Num 33:3, 5. In none of these cases is the formula “old name +הוא+ new name” used, nor does a longer explanatory gloss with the word לָרִאשֹׁנה—“at the first” occur with any of the five citations. In other words, there is no evidence within these five passages to suspect that “Rameses” is an editorial gloss.’, Hoffmeier, ‘What is the Biblical Date For the Exodus? A Response to Bryant Wood’, Journal of the Evangelical Theological Society, 50/2 (2007), 234.

[32] ‘Görg’s reading of this name as “Israel” is plagued by serious linguistic and orthographic problems that preclude it from being Israel.’, Hoffmeier, ‘What is the Biblical Date For the Exodus? A Response to Bryant Wood’, Journal of the Evangelical Theological Society, 50/2 (2007), 241.

[33] ‘Especially given the absence of Israel from the Armana evidence, this seems intrinsically unlikely, given the early date and lacking a full reading.’, Fleming, The Legacy of Israel in Judah’s Bible: History, Politics, and the Reinscribing of Tradition (2012), 241.

[34] ‘A close reading of the text indicates that God gave Israel victory over her oppressors in a major battle 25 miles away from Hazor, but the text is absolutely silent regarding any military action against Hazor itself. Furthermore, the terminology used in 4:23–24 is not found in Joshua or Judges to indicate attacks on cities. Consequently, there is no basis to believe that the destruction of the final LB IIB (late 13th century) city was caused by Deborah and Barak’s triumph over Jabin and Sisera’, Hoffmeier, ‘What is the Biblical Date For the Exodus? A Response to Bryant Wood’, Journal of the Evangelical Theological Society, 50/2 (2007), 244.

[35] ‘From the Amarna letters, written to the pharaohs Amernhotep III and Akhenaten between 1390–1340 BC, we learn that Hazor was thriving during this period.’, ibid., p. 245.

[36] Exodus 2:23 During that long period of time the king of Egypt died, and the Israelites groaned because of their slave labor. They cried out, and their desperate cry because of their slave labor went up to God.

[37] ‘Psalm 136:15 may be the closest to suggest that pharaoh drowned in the seas, but that may be due to misleading English translations, e.g. JB: “Drowned Pharaoh and his army”; NIV: “swept pharaoh and his army into the Red Sea”; KJV and NAS: “He overthrew Pharaoh. .. into the Red Sea.” The key word here is נאר, which is the word used in Exod 14:27. נאר, means to “shake off” (Ludwig Koehler and Walter Baumgartner, The Hebrew and Aramaic Lexicon of the Old Testament [Leiden: Brill, 2001] 707). Nothing in this term suggests that pharaoh drowned in the sea. In fact, there is nothing to suggest in the various texts, especially in Exodus, that pharaoh led the chariot corps in pursuit of the escaping Hebrews. Perhaps people have been influenced by Cecil B. DeMille’s portrait of angry Ramesses (Yul Brynner) leading the attack at the sea. But even in The Ten Commandments, Ramesses does not follow the Israelites into the sea!’, Hoffmeier, ‘What is the Biblical Date For the Exodus? A Response to Bryant Wood’, Journal of the Evangelical Theological Society, 50/2 (2007), 239.

[38] ‘The second problem for Wood’s exodus pharaoh drowning in the sea is that the mummy of Thutmose III was found in the Deir el-Bahri cache, while Amenhotep IIs was actually discovered in his tomb, one of only a few royal mummies discovered intact.81  In fact, all the mummies of the 15th century are accounted for.82  According to the X-rays and investigations of these mummies, none indicate a death by drowning.’, ibid., p. 240.

[39] ‘The need for discussing the latter premise is that many biblical scholars who affirm the historicity of the exodus now date it to the thirteenth century B.C., questioning concrete numbers in the Bible that taken literally would place the exodus in the fifteenth century B.C.’, Petrovich, ‘Amenhotep II And The Historicity Of The Exodus-Pharaoh’, The Master’s Seminary Journal, 17/1 (2006), 83.

[40] ‘Dating the period of the oppression and exodus to the fifteenth century B.C. has largely been replaced in favor of a thirteenth-century date, although a few adherents to the earlier date have followed Jack’s thesis.’, Hoffmeier, Israel In Egypt: The Evidence for the Authenticity of the Exodus Tradition (1999), 125.

h1

The Date of the Exodus (1/2)

February 19, 2015

Despite over a century of detailed investigation, the date of the Hebrew exodus from Egypt remains a topic of extensive debate within scholarship.[1] Scholarly discussion focuses on exegetical concerns such as the interpretation of chronological data in the Old Testament, the correct identification of toponyms (place names), and the relationship between textual and archaeological data. This initial article summarizes the emergence of the two most commonly proposed dates.

History of Interpretation

Throughout the 19th century, Rameses II was considered the pharaoh under whom the Hebrews were enslaved, and his son Merneptah the pharaoh of the exodus.[2]  However, discovery of the Merneptah Stele, referring to Israel as a recognized people settled in Canaan by the 14th century, invalidated this view.

‘This new data appeared to require that Israel had already been settled there by the end of the 13th century BC. Placing Israel in Canaan this early in the reign of Merneptah raised obstacles for his having been the pharaoh of the Exodus. Israel obviously could not have left Egypt in the first year of Merneptah’s reign, wandered in the wilderness for forty years, and then appeared in Canaan as a settled ethnic group in his fifth year.’[3]

A revised interpretation identified Ramases II as the pharaoh of the exodus, a view which remained dominant throughout the rest of the 19th century, up to the 1920s.[4] [5] In 1925 a 15th century date was proposed.

‘This approach seems to have been pioneered initially by James Jack, who challenged the 13th-century BC date in his 1925 book, The Date of the Exodus in the Light of External Evidence. Jack argued that both biblical and extrabiblical evidence pointed to a mid-15th century BC date.’[6]

Textual Evidence

The early date rests principally on an application of the chronology given in 1 Kings 6:1, which appears to date the exodus 480 years before the reign of Solomon.[7] Since there is considerable agreement that Solomon’s reign started at around 960-960 BCE, counting 480 years back from this date places the exodus at c.1440 BCE. A second text seen as corroborating the early date is Judges 11:26, in which Jephthah’s claim that Israel had already been in Canaan for at least three hundred years[8] would appear to suggest an early date for the exodus.[9]

The late date receives textual support from Exodus 1:11, which refers specifically to the Hebrews building Pithom and Rameses for the Pharaoh.[10] Unlike 1 Kings 6:1 and Judges 11:26, this text rests on an absolute rather than a relative date; the construction of buildings at Pithom and Rameses.

‘Late-date theorists argue that, since the Exodus account used the name by which the city was known for about two centuries only (c. 1300–1100 B.C.), the Hebrew tradition of the exodus must also date from that period. In such an event Rameses II would have been the pharaoh of the oppression, and his son Merneptah (1224?–1214 B.C.) the pharaoh of the exodus.’[11]

Archaeological Evidence for the Conquest

In the absence of direct archaeological evidence for the Hebrew settlement in Goshen, and the lack of Egyptian records describing the Hebrews as an enslaved ethnic group, or the plagues, or subsequent exodus, attempts to date the exodus using archaeological evidence focused on dating the Hebrew entry into Canaan, searching for evidence of conquest.

Attempts have been made by proponents for both dates, and interpretation of the archaeological record has been much contested. In the 1930s, archaeologist John Garstang’s excavations of Tell es-Sultan led him to conclude there was strong evidence for a Hebrew destruction of Jericho before 1400 BCE, lending weight to an early date exodus.[12]

However, Kathleen Kenyon’s subsequent investigation of the site re-dated the destruction to around 1500 BCE, too early for the Hebrews.[13] Efforts by Bryant Wood to defend Garstang’s dating and attribute the destruction of Jericho to an early date Hebrew conquest,[14] [15] have not gained any significant scholarly acceptance outside evangelical circles. Contemporary with Garstang, archaeologist Wiliam Albright arrived at a late date for the exodus, on the basis of his investigations of Canaanite archaeological sites.[16]

In response to acknowledged difficulties harmonizing an early date with the archaeological record, in the 1970s John Bimson proposed a Hebrew conquest during the end of the Middle Bronze Age, which seemed to fit the Biblical record more closely.[17] However, Bimson’s interpretation was critiqued strongly by many scholars.

‘The critique of Bimson’s proposal came from numerous quarters. Bietak objected that his suggested alteration was only fifty years, therefore still in the sixteenth century, and could not be stretched as far as Bimson needed it to be. B. Halpern objected that the changes suggested by Bimson would leave a reduced time span for LB I that could not possibly accommodate the archaeological data.’ [18]

Archaeological evidence for destruction and occupation layers supporting a late date exodus, was considered more abundant.

‘Archaeological evidence from Canaanite sites such as Bethel, Debir, Lachish, and Hazor indicates destruction at 13th-century B.C. levels, a fact generally regarded as relating to the Hebrew occupation under Joshua.’[19]

Evidence from Philistine sites tends to favor a late rather than an early date for the exodus. Against that, however, must be set the fact that the major Philistine occupation of the southern Palestinian coastlands only occurred around 1175 B.C., in the time of Rameses III.’[20]

The Ongoing Dispute

By the 1970s the date of the exodus had ceased to become a significant concern within critical scholarship, as many commentators no longer believed in the essential historicity of the event. However, the issue continued to be debated hotly among evangelical and other faith professing scholars, as well as among a minority report of critical scholars and those professional archaeologists who considered the Biblical exodus account to preserve an essentially accurate historical core.

The second article in this series will compare and contrast the evidence and arguments advanced for each date, together with their respective counter-arguments.


[1]The date of the Exodus is one of the most debated topics in OT studies because of the ambiguous nature of the evidence.’, Shea, ‘Exodus, Date of the’, in Bromiley et al. (eds.), The International Standard Bible Encyclopedia, Revised, volume 2 (1979–1988), 230.

[2] ‘With the identification of Ramesses II as the pharaoh of the oppression, his son Merneptah, who succeeded him on the throne, naturally became the pharaoh of the Exodus. Based on this reasoning, the biblical Exodus was securely located by scholars within the 19th Dynasty of Egypt (1293–1185 BC) throughout the nineteenth century.’, Hawkins, ‘Propositions For Evangelical Acceptance Of A Late-Date Exodus-Conquest:  Biblical Data And The Royal Scarabs From Mt. Ebal’, Journal of the Evangelical Theological Society, 50/1 (2007), 31-32.

[3] Ibid., p. 32.

[4] ‘Up until about 1925, this position was widely held by scholars, both evangelical and otherwise.’, ibid., p. 32.

[5]At the beginning of the 20th century many scholars, both liberal and conservative, placed the date toward the end of the 13th century B.C.’, Harrison, ‘Exodus, The’, in Elwell & Beitzel, Baker Encyclopedia of the Bible (1988), 742.

[6] Hawkins, ‘Propositions For Evangelical Acceptance Of A Late-Date Exodus-Conquest:  Biblical Data And The Royal Scarabs From Mt. Ebal’, Journal of the Evangelical Theological Society, 50/1 (2007), 33.

[7] 1 Kings 6:1 In the four hundred and eightieth year after the Israelites left Egypt, in the fourth year of Solomon’s reign over Israel, during the month Ziv (the second month), he began building the LORD’s temple.

[8] Judges 11:26 Israel has been living in Heshbon and its nearby towns, in Aroer and its nearby towns, and in all the cities along the Arnon for three hundred years! Why did you not reclaim them during that time?

[9] ‘If 1100 BC is taken as an approximate date for Jephthah’s activities, this would place the taking of the Transjordan under Moses (Numbers 21) around 1400 BC, about 40 years after the departure from Egypt.’, Hawkins, ‘Propositions For Evangelical Acceptance Of A Late-Date Exodus-Conquest:  Biblical Data And The Royal Scarabs From Mt. Ebal’, Journal of the Evangelical Theological Society, 50/1 (2007), 33.

[10] Exodus 1:11 So they put foremen over the Israelites to oppress them with hard labor. As a result they built Pithom and Rameses as store cities for Pharaoh.

[11] Harrison, ‘Exodus, The’, in Elwell & Beitzel, Baker Encyclopedia of the Bible (1988), 743.

[12] ‘Garstang identified several levels of debris there, indicating that the city had been rebuilt a number of times. He concluded that the one built about 1500 B.C. was the Jericho overthrown by Joshua’s forces (Jos 6). Garstang’s statement that Jericho had fallen before 1400 appeared to support the time frame of 1 Kings 6:1, and was received enthusiastically by supporters of the 15th-century B.C. date.’, ibid., p. 743.

[13] ‘Garstang’s discoveries at Jericho have been modified seriously by the subsequent work of another archaeologist, Kathleen Kenyon. She found no trace of Late Bronze Age walls, which indicates that the city Garstang thought to have been conquered by Joshua was considerably earlier than his time. Unfortunately, the mound has been so ravaged by erosion and human pillaging that it reveals almost nothing about the Jericho of Joshua’s day, and thus does not help to simplify matters.’, ibid., p. 744.

[14] Wood, ‘Did the Israelites Conquer Jericho? A New Look at the Archaeological Evidence’, Biblical Archaeology Review, 16/2 (March/April 1990), 44-58.

[15] Wood, ‘The Walls of Jericho’, Bible and Spade 12/2 (1999).

[16] ‘The 13th-century exodus-conquest theory was formulated by William F. Albright in the 1930s, based largely on Palestinian archaeological evidence, and promoted by him throughout his career.’, Wood, ‘The Rise and Fall of the 13th-Century Exodus-Conquest Theory’, Journal of the Evangelical Theological Society, 48/3 (2005), 473.

[17] ‘The Late Bronze Age (LB) was characterized by a problematic lack of fortified cities. The MB in Canaan, in sharp contrast, featured the massive and numerous walled cities that the books of Numbers and Joshua seem to suggest, and many of them were destroyed at the end of the period.’, Walton, ‘Exodus, Date of’, in Alexander & Baker, Dictionary of the Old Testament: Pentateuch (2003), 259.

[18] Ibid., p. 259.

[19] Harrison, ‘Exodus, The’, in Elwell & Beitzel, Baker Encyclopedia of the Bible (1988), 744.

[20] Ibid., p. 744.

h1

Failed prophecies?

July 3, 2014

Prophecies are consistently appealed to in the Bible as evidence of its divine origin. Several passages claimed by skeptics and atheists as examples of failed prophecies are examined here. They fall into the following categories: passages which are not formal predictive prophecies in the manner claimed, passages which are misinterpreted by the critic, and passages which were fulfilled in contradiction to the critic’s claims.

Passages which are applied typologically

Several passages in the Old Testament which have  traditionally been understood as prophetic of Christ, are objected to by critics as inapplicable or unfulfilled.

  1. How can ‘When he sins, I will correct him with the rod of men and with wounds inflicted by human beings’ in 2 Samuel 7:14 apply to Christ, when Christ never sinned?
  1. How can the servant of Isaiah 49 refer to Christ, when the context indicates it speaks explicitly of Israel?

A traditional answer has been that these prophecies have ‘dual fulfillments’, that they were prophecies fulfilled in part by Solomon or Israel, and fulfilled in part later in the life of Christ. This is actually only half true. In reality the prophecies had direct and full application to their immediate referent, and are applied typologically to Christ. That is, they are formal predictive prophecies about Solomon and Israel respectively, but parts of them find echoes in the life of Christ, the ‘son of God’ in a greater sense than both Solomon and Israel. Their application to Christ is not a claim to direct fulfillment of a formal prediction, but an illustration that certain promises made by God in earlier times have a relevance to the greater work of Christ which they foreshadow.

Passages which are misinterpreted

Certain prophecies criticized as unfulfilled have actually been misinterpreted.

  1. How can God’s words  ‘I, the LORD, promise: “David will never lack a successor to occupy the throne over the nation of Israel’ in Jeremiah 33:17 be said to have been fulfilled, when the Davidic monarchy ended with the Babylonian captivity?

The answer is that this prophecy was conditional, as was made clear to Solomon before Jeremiah’s time.

1 Kings 9:

You must serve me with integrity and sincerity, just as your father David did. Do everything I commanded and obey my rules and regulations.

Then I will allow your dynasty to rule over Israel permanently, just as I promised your father David, ‘You will not fail to have a successor on the throne of Israel.’

Solomon and his descendants failed to keep this covenant, and the Davidic monarchy consequently came to an end;[1] the New Testament writers identify Christ as its true successor.

  1. How can Jesus’ words ‘I tell you the truth, you will not finish going through all the towns of Israel before the Son of Man comes’ in Matthew 10:23 and ‘this generation will not pass away until all these things take place’ in Matthew 24:34 be considered anything but failed prophecies, when he did not return before the apostles had finished preaching to the towns of Israel, or before the generation of his time had passed away?

These passages do not speak of the return of Christ, but of the end which would come with the destruction of Jerusalem in the war of 66-70 CE.[2] [3]

  1. In Mark 8:38 Jesus says ‘there are some standing here who will not experience death before they see the kingdom of God come with power’, and in Matthew 16:28 Jesus says ‘there are some standing here who will not experience death before they see the Son of Man coming in his kingdom’, Jesus failed to return before his disciples died, making these false prophecies.

These passages refer to thetransfiguration (a vision of Jesus ‘coming in his kingdom’), which is described in both gospels as occurring shortly after Jesus spoke these words. Early Christians almost universally understood these passages as a reference to the transfiguration.[4]

It should be noted that these words of Jesus were transmitted by the gospel writers decades after Jesus’ ministry and after the disciples had died, indicating that they them as fulfilled prophecy. If these predictions had been understood as Jesus saying he would return before the disciples died, they would have been more likely to omit them completely.

  1. In 1 Corinthians 7:31 Paul says ‘the present shape of this world is passing away’, indicating that he, like other New Testament writers, believed they would see the return of Christ in their lifetime, but it never happened.

These words are not written as formal predictive prophecy, and as much as Paul may have believed at one time that Christ would return while he was still alive,  it is clear he and other writers such as Peter gradually understood that this would not be the case. In fact both Paul and Peter prepare other Christians for a lengthy wait before Jesus’ return. In 2 Thessalonians Paul makes it clear that Christ’s return is not imminent, and that it will not happen before certain specific events have taken place.[5]

2 Thessalonians 2:

1 Now regarding the arrival of our Lord Jesus Christ and our being gathered to be with him, we ask you, brothers and sisters,

2 not to be easily shaken from your composure or disturbed by any kind of spirit or message or letter allegedly from us, to the effect that the day of the Lord is already here.

3 Let no one deceive you in any way. For that day will not arrive until the rebellion comes and the man of lawlessness is revealed, the son of destruction.

Peter likewise prepares his readers for a lengthy wait, speaking of the ‘last days’ in the future, and warning believers they will be mocked for their patience at that time.

2 Peter 3:

3 Above all, understand this: In the last days blatant scoffers will come, being propelled by their own evil urges

4 and saying, “Where is his promised return? For ever since our ancestors died, all things have continued as they were from the beginning of creation.”

Passages which were fulfilled

Certain prophecies have actually been fulfilled, despite claims to the contrary.

  1. The prophecy of Isaiah 17 claims Damascus would be ‘a heap of ruins (verse 1), and that ‘Damascus will lose its kingdom’ (verse 23), but Damascus is a thriving city today.

This prophecy made by Isaiah some time around 740 BCE, was fulfilled when the Assyrian king Tiglath-pileser III destroyed Damascus, which was at that time not a mere city but a thriving Aramean kingdom.[6] Isaiah prophesied Damascus would be a ‘heap of ruins’, and Tiglath-pileser III’s own record of his conquest proves this is what happened, boasting of his extensive destruction of Damascus; ‘591 cities of the 16 districts of Damascus I destroyed like mounds of ruins after the Deluge’. [7] Isaiah’s prediction that ‘Damascus will lose its kingdom’ was fulfilled by Tiglath-pileser III’s annexation of Damascus and all its territories, and the deportation of  many of its inhabitants.[8] [9]

  1. The prophecy of Tyre’s destruction in Ezekiel 26 failed to come true, and Ezekiel’s own words in Ezekiel 29:18-19 (especially that Nebuchadnezzar II and his army ‘received no wages from Tyre for the work he carried out against it’ , verse 28), prove he knew the prophecy failed.

This criticism overlooks the fact that Ezekiel’s commentary in Ezekiel 29:18-19 is certainly in response to criticisms that his earlier prophecy was not completely successful; in other words, it proves at minimum that Ezekiel’s earlier prediction that Nebuchadnezzar II would attack Tyre was not only made before the event, but also came true. This is not a good start for a critic of prophetic fulfillment. Subsequent to Ezekiel’s successful prediction, his political enemies attempted to discredit him by claiming Nebuchadnezzar’s army had failed to  vindicate the prophecy.[10] Such claims were unwarranted, for ‘the siege was successful and Tyre did pass into Babylonian control’.[11]

Since Nebuchadnezzar II failed to destroy Tyre utterly, some commentators state that since Ezekiel’s original prophecy had predicted ‘many nations’ would be brought against Tyre (‘I will bring up many nations against you’, Ezekiel 26:3), of which Nebuchadnezzar II’s campaign was only the first. It is claimed that this is supported by the alternating statements of what ‘he’ (Nebuchadnezzar II), and ‘they’ (subsequent nations), would do to Tyre, especially Alexander the Great.

However, this view has been criticized as an improbable reading of the Hebrew text.[12] Instead it should be recognized that the first section of the prophecy (verses 1-6), is a self-contained unit predicting the coming of ‘many nations’ against Tyre, and thus not restricted to Nebuchadnezzar II (whose campaign is described from verses 7-13);[13] the complete destruction of the city as predicted by Ezekiel was fulfilled by Alexander.

The objection that the prophecy’s description of an attack against a mainland city (which Nebuchadnezzar attacked), and therefore does not describe the destruction of the island (which Alexander attacked), is misguided. [14] The prophecy uses the standard conquest language of the Ancient Near East; when Esarhaddon of Assyria attacked the island city he still described it in terms of a land battle, even to the point of describing trenches being dug (impossible in an island siege).[15]

  1. The prophecies against Egypt in Ezekiel 30 and Isaiah 19 failed to come true; Nebuchadnezzar did not invade and conquer Egypt as predicted.

First it should be noted that it is acknowledged even by modern critics that the prophecy was given before the event it describes took place.[16] Consequently, attempts to reduce the accuracy of the prediction by claiming it was insufficiently fulfilled are demonstrably motivated by the desire to avoid the fact that a successful prophecy actually happened. The best a critic can do in the face of the fact that the text contains a prophecy indisputably before the event to which it obviously refers, is to claim that the prophecy wasn’t fulfilled sufficiently to be considered accurate. This is not intellectually honest.

In fact, the prophecy was fulfilled by Nebuchadnezzar II’s war against Egypt in 586 BCE, recorded in a fragmentary Babylonian text.

‘. . . [in] the 37th year, Nebuchadnezzar, king of Bab[ylon] mar[ched against] Egypt (Mi-sir) to deliver a battle. [Ama]sis (text: [ . . . ]-a(?)-su)y of Egypt, [called up his a]rm[y] . . . [ . . . ]\u from the town Pufu-Iaman . . . distant regions which (are situated on islands) amidst the sea . . . many . . . which/who (are) in Egypt . . . [car]rying weapons, horses and [chariot]s . . . he called up to assist him and . . . did [ . . . ] in front of him . . . he put his trust. . . (only the first signs at the beginning and the end of the following 7 or 8 lines are legible).’[17]

The success of Nebuchadnezzar’s campaign is acknowledged by secular historians, on the basis of several lines of evidence.[18] Firstly, and rather ironically, some historians believe the prophecy of Ezekiel was written after the event, precisely because it is so accurate.

‘First of all, both Ezekiel and Jeremiah prophesied that he would do so; and since most of these “prophecies” were written in retrospect, or at least gained popular currency only after having been proved correct, we may be fairly certain that the prophesied invasion and defeat of Egypt actually took place.’[19]

Two other sources are the Biblical text describing Jewish refugees moved from Egypt to Babylon, and the record of Josephus indicating Nebuchadnezzar defeated Egypt.

‘Secondly, the biblical sources say that Nebuchadrezzar was able to remove the Jewish refugees in Egypt to Babylon. He could not of course have done so unless he had entered and subjugated the country. Thirdly, Josephus tells us that he conquered Egypt. We are informed that four years after the fall of Tyre, Nebuchadrezzar invaded the country and put its King Uaphris to death, installing a creature of his own upon the vacant throne.’[20]

Another source is the presence in Egypt of artifacts belonging to Nebuchadnezzar II, demonstrating he invaded and established himself there.

‘Fourthly, and most importantly, artifacts of Nebuchadrezzar have actually been discovered in Egypt. These are “three cylinders of terra-cotta bearing an inscription of Nebuchadnezzar, an ordinary text referring to his constructions in Babylon … These were said to come from the Isthmus of Suez, and they apparently belong to some place where Nebuchadrezzar had ‘set up his throne’ and ‘spread his royal pavilion.’ As he only passed along the Syrian road, and Daphnae would be the only stopping place on that road in the region of the isthmus, all the inferences point to these having come from Defenneh, and being the memorials of establishment there.”’ [21]

This also fulfills the prophecy of Jeremiah 43:10 that Nebuchadnezzar would ‘pitch his royal tent’ in Tahpanhes in Egypt.[22] [23] The scholarly  conclusion from these sources is ‘There can be little doubt; Nebuchadrezzar entered and conquered Egypt.’ [24]

  1. Although Joshua 3:10 says God would ‘truly drive out before you the Canaanites, Hittites, Hivites, Perizzites, Girgashites, Amorites, and Jebusite’, 1 Kings 9:20 says ‘several non-Israelite peoples were left in the land after the conquest of Joshua’.

Joshua’s words are a repetition of the words of Moses, which made it clear that God’s driving out of the  inhabitants of Canaan was conditional on the Hebrews maintaining their obedience to God.

Deuteronomy 4:

1 Now, Israel, pay attention to the statutes and ordinances I am about to teach you, so that you might live and go on to enter and take possession of the land that the LORD, the God of your ancestors, is giving you.

Deuteronomy 7:

12 If you obey these ordinances and are careful to do them, the LORD your God will faithfully keep covenant with you as he promised your ancestors.

Deuteronomy 8:

1 You must keep carefully all these commandments I am giving you today so that you may live, increase in number, and go in and occupy the land that the LORD promised to your ancestors.

  1. The gospels describe Jesus predicting the destruction of Jerusalem and the Temple, but this is not an accurate prediction because the gospels were written after the event; even if they had been written before the event, such a war was obviously going to happen anyway, so it is not evidence of an accurate prophecy.

The first point to note about this objection (which can be found made by a range of atheists and skeptics), is that it is intellectually dishonest; it tries to argue that the prophecy isn’t true because it was written after the event, but also argues that even if it had been made before the event it still wouldn’t count as a fulfilled prophecy. Such an argument is not evidence based, and demonstrates that the person making the argument is not really interested in the facts.

The second point to note about this objection is that although the date of Mark’s gospel is still an open question in current scholarship, dates proposed typically fall between 65 and 75 CE.[25] In fact recently strong arguments have been made for a much earlier date. The secular scholar Mark Crossley argues for a date ‘before the late forties’,[26] at least 20 years before the destruction of Jerusalem, and the secular scholar Maurice Crossley argues ‘a date c. 40 CE must be regarded as highly probable’.[27]

It is clear that many scholars (even non-religious scholars), are prepared to accept that Mark’s gospel (which contains a prophecy of the destruction of Jerusalem and the Temple), was indeed written before the events it predicts. Rejecting this possibility out of hand is therefore intellectually dishonest, especially when no attempt is made to address the significant body of evidence indicating that Jesus’ prediction in Mark was indeed made before the event.

The third point to note is that there is no evidence at all to suggest that the possibility of the destruction of Jerusalem and the Temple by the Romans was so probable that it would have appeared obvious to people in Jesus’ own day, or even 30 years later (63-65 CE). In fact construction on the Temple had already been carried out for nearly 50 years by the time of Jesus’ ministry (John 2:20), and it was not even completed until shortly before the Romans destroyed it.[28][29]

It is difficult to demonstrate that anyone seeing the ongoing construction of the Temple in Jesus’ day would have concluded that the Romans would destroy it over 30 years later. It is likewise difficult to demonstrate that anyone seeing the construction continuing in 63 CE would have concluded that the Romans would destroy both the Temple and the entire city in just a few more years.

The fact is that the explosive events which led to the destruction of Jerusalem in 70 CE were not predicted by anyone outside the gospels, and even Jesus’ own words make it clear he expects skepticism on the part of his audience. There is no evidence in any of the relevant Roman historical sources who wrote in detail about the events of the first century (such as Suetonius, Tacitus, and Appian), nor in Josephus (who was both a historian, and a general on the Jewish side of the war), that anyone was expecting such a war before it took place, still less the complete destruction of Jerusalem and the Temple. There is excellent evidence therefore that Jesus’ prophecy was made well before the events they predicted successfully, and it is indisputable that these events were not foreseen or expected by anyone else.

Conclusion

Skeptical claims that Bible prophecies have not been fulfilled should be taken seriously; it is a fact that some Bible prophecies are impossible to verify due to a lack of available information, even if there is no evidence proving they did not come to pass. However, when such criticisms are made it is important to identify whether or not the arguments made are evidence based, demonstrate a knowledge of and engagement with the relevant scholarly literature, and are intellectually honest.

In particular, such arguments must prove that they have interpreted the prophecy as it was originally intended, and must provide substantial objections to the relevant counter-arguments by scholars who make the case that the prophecy was in fact fulfilled. In turn, defenses of these prophecies must exercise intellectual honesty in acknowledging problems where they genuinely exist, and must cite, discuss, and be supported by the relevant scholarly literature, in order to be credible.

____________________________________

[1] ‘But perhaps the promise to the house of David is not all that unilaterally unconditional, and perhaps the dynastic oracle here is firmly set in the Sinaitic covenant (Eslinger 1994). We can agree with Brueggemann (1990: 259) when he says, “While the covenantal ‘if’ is silenced in this theology, it has not been nullified.” This is particularly true when the Lord speaks of David’s son: “When he commits iniquity, I will punish him with a rod such as mortals use, with blows inflicted by human beings” (v. 14b).’, Victor P. Hamilton, Handbook on the Historical Books (Grand Rapids, MI: Baker Academic, 2001), 317–318.

[2]Now that we have seen that the reference is to the destruction of the temple, which did as a matter of fact take place some 40 years later while many of Jesus’ contemporaries must have been still alive, all such contrived renderings may be laid to rest. This verse refers to the same time-scale as 16:28 (which was also concerned with the fulfillment of Dan 7:13–14): “some of those standing here will certainly not taste death before …” (cf. also 10:23, with the same Daniel reference: “you will not go through all the towns of Israel before …”).’, R. T. France, The Gospel of Matthew (The New International Commentary on the New Testament; Grand Rapids, MI: Wm. B. Eerdmans Publication Co., 2007), 930.

[3]34 The πάντα ταῦτα, “all these things,” of this verse can include no more than the same phrase in the preceding verse and thus cannot include the coming of the Son of Man (so too Blomberg). The phrase refers not only to general marks of the interim period such as tribulation, distress, pseudo-messiahs, and false prophets but specifically, and dramatically, to the desecration of the temple and the destruction of Jerusalem (cf. vv 15–22). As in the other imminence sayings (cf. 16:28; 10:23; 23:36), all of which like the present logion are prefaced by the emphatic ἀμὴν λέγω ὑμῖν, “truly I tell you,” formula, the main point is that the fall of Jerusalem was to be experienced by that generation (pace Kidder), those listening there and then to the teaching of Jesus (ἡ γενεὰ αὕτη, “this generation,” is used consistently in the Gospel to refer to Jesus’ contemporaries; cf. 11:16; 12:41–42, 45; 23:36).’ Donald A. Hagner, Matthew 14–28 (vol. 33B; Word Biblical Commentary; Dallas: Word, Incorporated, 1998), 715.

[4]‘The most widespread interpretation in the Eastern and Western church related the saying to the transfiguration whereby then “some” referred to Peter, James, and John.’, Ulrich Luz, Matthew: a Commentary (ed. Helmut Koester; Hermeneia—a Critical and Historical Commentary on the Bible; Minneapolis, MN: Augsburg, 2001), 386; Luz disagrees with this interpretation, but acknowledges it was the most common in the early church.

[5] ‘Mearns (“Development”) also takes the reference to be to 1 Thessalonians, but argues that Paul had changed his mind about the suddenness of the Day of the Lord since writing that letter and is now correcting perfectly reasonable inferences that the readers might have drawn from it.’, F. F. Bruce, 1 and 2 Thessalonians (vol. 45; Word Biblical Commentary; Dallas: Word, Incorporated, 1998), 164.

[6] ‘The Assyrian invaded northern Israel (2 K. 15:29) and then besieged and destroyed Damascus as an Aramean kingdom in 732, killing Rezin (2 K. 16:9). Assyrian records tell of 591 towns of the “16 districts of Aram” destroyed “like mounds left by a flood” (ARAB, I § 777).’, M. F. Unger, “Damascus,” ed. Geoffrey W. Bromiley, The International Standard Bible Encyclopedia, Revised (Wm. B. Eerdmans, 1979–1988), 854.

[7] ‘12′ […] orchards without number I cut down; I did not leave a single one. 13′ … the town of …]hadara, the home of the dynasty of Rezin of Damascus, 14′ [the pl]ace where he was born, I surrounded and captured. 800 people with their possessions 15′ their cattle (and) their sheep I took as spoil. 750 captives from the cities of Kuruṣṣa 16′ (and) Sama, 550 captives from Metuna I took, 591 cities 17′ of the 16 districts of Damascus I destroyed like mounds of ruins after the Deluge.’, Lester L. Grabbe, “The Kingdom of Israel from Omri to the Fall of Samaria: If We Only Had the Bible …,” in Ahab Agonistes: The Rise and Fall of the Omri Dynasty (ed. Lester L. Grabbe; London: T&T Clark, 2007), 79.

[8] ‘Finally, the Assyrians had had enough of the rebellious behaviour of Damascus, the last Aramaean stronghold in Hatti. Damascus and its cities were conquered and turned into Assyrian provinces (Tadmor 1994: 79–81). A part of the population was deported (Grayson 1991/2000: 77–8; Dion 1997: 215–16; Sader 1987: 250–1; Weippert 1987: 99).’, Hans M. Barstad, “Can Prophetic Texts Be Dated? Amos 1–2 as an Example,” in Ahab Agonistes: The Rise and Fall of the Omri Dynasty (ed. Lester L. Grabbe; London: T&T Clark, 2007), 33.

[9] ‘At the close of the Syro-Ephraimitic War, Tiglath-pileser took several actions that form the background of this text. He killed Rezin, destroyed Damascus, and annexed all territory controlled by Damascus into the Assyrian provincial system.‘, Brad E. Kelle, “What’s in a Name? Neo-Assyrian Designations for the Northern Kingdom and Their Implications for Israelite History and Biblical Interpretation,” ed. Gail R. O’Day, Journal of Biblical Literature 121 (2002): 639., Gail R. O’Day, ed., Journal of Biblical Literature 121 (2002): 659.

[10] ‘The objective reason for the oracle is supplied in v 18. However, the more immediate agenda is implied by v 21ab: Ezekiel was being criticized by his Jewish contemporaries for the lack of precise fulfillment of his oracles against Tyre.’. Leslie C. Allen, Ezekiel 20–48 (vol. 29; Word Biblical Commentary; Dallas: Word, Incorporated, 1998), 109.

[11] ‘It was to some extent a carping criticism: the siege was successful and Tyre did pass into Babylonian control. In a list of royal hostages at Nebuchadnezzar’s court, to be dated about 570 B.C., the king of Tyre has the initial place (ANET 308a; Katzenstein, History of Tyre 326). About 564 B.C. Baal, Ethbaal’s successor as king of Tyre, was replaced by a Babylonian High Commissioner (Katzenstein, History 332–33; cf. Unger, ZAW 44 [1926] 314–17). Any prophet might have been glad to chalk it up as a vindication of his or her prediction, despite Nebuchadnezzar’s non-destruction of Tyre.’, ibid., p. 109.

[12] ‘However, the proposed distinction between the “many nations” of verse 3 and Nebuchadnezzar’s army seems overly subtle, in light of the reference to Nebuchadnezzar as “king of kings” (v. 7) and the multiethnic nature of his army. Nebuchadnezzar is the focal point of verses 7–11, but the actions described are those of an army. The subject of the plural forms in verse 12 is most naturally understood as the collective “army” (Heb. ‘am) of verse 7, which in turn can be seen as comprised of the “many nations” mentioned in verse 3 (see also the reference to “nations” in v. 5).’, Robert B. Chisholm Jr., Handbook on the Prophets: Isaiah, Jeremiah, Lamentations, Ezekiel, Daniel, Minor Prophets (Grand Rapids, MI: Baker Academic, 2002), 268.

[13]Tyre’s fall appears in the first two prophecies, the second picking up phrases from the first and adding further details, and the remaining two prophecies describe the bewailing and then the entombment of the fallen one.’, Ronald M. Hals, Ezekiel (vol. 19; The Forms of the Old Testament Literature; Grand Rapids, MI: William B. Eerdmans Publishing Company, 1989), 188.

[14]Taking the standard siege imagery too literally, some scholars have concluded that the passage must come out of a setting different from Nebuchadrezzar’s time, seeing in vv. 9–11 a battle song about Alexander’s conquest of Tyre, or supposing that the actual reference of the passage was to a conquest of “old Tyre” on the adjacent mainland. (See the references in Zimmerli, Ezekiel 2, 37.) The proper recognition of the typicality of the imagery involved renders such literalistic hypotheses unnecessary.’, Ronald M. Hals, Ezekiel (vol. 19; The Forms of the Old Testament Literature; Grand Rapids, MI: William B. Eerdmans Publishing Company, 1989), 189.

[15] ‘The infiltration of characteristics typical of a description of a mainland siege into the description of the siege of the island city of Tyre can already be observed in the Assyrian royal inscriptions, when Esarhaddon there reports of the construction of trenches (ḫalṣē) against Tyre.’, Walther Zimmerli, Frank Moore Cross, and Klaus Baltzer, Ezekiel: a Commentary on the Book of the Prophet Ezekiel (Hermeneia—a Critical and Historical Commentary on the Bible; Philadelphia: Fortress Press, 1979–), 37; a footnote adds ‘This also disposes of Wiener’s theory that this passage deals solely with the conquest of the mainland city of old Tyre’.

[16] ‘The prophecy against Egypt does not seem to have come to pass in every detail either, but the book was probably completed and its authority established by the time this became clear.’, Thomas Renz, The Rhetorical Function of the Book of Ezekiel (Leiden: Brill, 2002), 98.

[17] J. B. Pritchard (ed.), Ancient Near Eastern Texts (3rd ed. with supplement. Princeton: Princeton UP, rev. 1969), 308.

[18] ‘That Nebuchadrezzar actually conquered Egypt is suggested by a number of very powerful pieces of evidence’, Emmet John Sweeney, The Ramessides, Medes, and Persians, Ages In Alignment Series, volume 4 (Algora Publishing, 2008), 153; ‘Nebuchadrezzar’ is the more accurate transliteration of the name ‘Nebuchadnezzar’.

[19] Ibid., p. 153.

[20] Ibid., p. 153.

[21] Ibid., p. 153.

[22] Jeremiah 43: 8 At Tahpanhes the LORD spoke to Jeremiah. 9 “Take some large stones and bury them in the mortar of the clay pavement at the entrance of Pharaoh’s residence here in Tahpanhes. Do it while the people of Judah present there are watching. 10 Then tell them, ‘The LORD God of Israel who rules over all says, “I will bring my servant King Nebuchadnezzar of Babylon. I will set his throne over these stones which I have buried. He will pitch his royal tent over them.

[23] ‘In short, the prophecy of Jeremiah that the king of Babylon would spread his royal pavilion at the entrance of the pharaoh’s house in Tahpanheth (Daphnae) was fulfilled.’, ibid., p. 153.

[24] Ibid., p. 153.

[25] ‘While scholars differ over the precise year, a date between 65 and 75 CE is accepted by a wide variety of scholars of very different ideological persuasions.’, James G. Crossley, The Date of Mark’s Gospel: Insight from the Law in Earliest Christianity (vol. 266; Journal for the Study of the New Testament Supplement Series; London; New York: T&T Clark International, 2004), 1.

[26] ‘This now becomes an argument of powerful collective weight for Mark to have been written before the late forties and if this is combined with the analysis of Mark 13 in Chapter 2 it is unlikely that it was written no earlier than the mid to late thirties.’, ibid., p. 208.

[27] Maurice Casey, Aramaic Sources of Mark’s Gospel (vol. 102;  Society for New Testament Studies; Cambridge University Press, 1998), 260.

[28] ‘According to Josephus,17 the Herodian temple was begun in 20/19 BCE; it was completed shortly before the war with Rome.18 If this scene may be used to date the events,19 it would have taken place on Passover of the year 28.’, Ernst Haenchen, Robert Walter Funk, and Ulrich Busse, John: a Commentary on the Gospel of John (Hermeneia—a Critical and Historical Commentary on the Bible; Philadelphia: Fortress Press, 1984), 184.

[29] ‘Work was still going on at his [Herod’s] death, and for that matter, for long after. The Temple was not completed until A.D. 63.’, Leon Morris, The Gospel According to John (The New International Commentary on the New Testament; Grand Rapids, MI: Wm. B. Eerdmans Publishing Co., 1995), 176.

h1

New book available: Living On The Edge

October 22, 2013

Living On The Edge: a book for doubting Christians

Today Christians in the Western world are typically living in a post-Christian society. Christian beliefs are met with skepticism, and people see little reason to believe. Christians are confronted with daily challenges to their faith, and often struggle to understand the relevance of Christianity to modern life. Professional surveys indicate the following reasons why young Christians lose their faith.

  • Overprotective churches
  • Shallow church experience
  • Antagonism towards science
  • Simplistic teaching on morality
  • Christianity seems exclusive
  • Not treating doubters kindly

This 600 page book (written in English), addresses those concerns, providing evidence upholding and defending Christian beliefs and values, and proving they are relevant to the modern world. It is aimed at Christians struggling with faith and re-assessing their beliefs, as well as Christians who are interested in building a stronger faith. It is also useful for Christians who want a book to show their non-Christian friends that the Christian faith is reasonable.

h1

Living On The Edge: challenges to faith

September 1, 2013

Today Christians in the Western world are typically living in a post-Christian society. Christian beliefs are met with skepticism, and people see little reason to believe. Christians are confronted with daily challenges to their faith, and often struggle to understand the relevance of Christianity to modern life.

The book ‘Living On The Edge: challenges to faith‘ (due to be printed in November 2013), addresses those concerns. For an overview of the book, click here.

h1

What is the Dunning-Kruger effect?

February 28, 2012

In 1999, David Dunning and Justin Kruger published a journal article entitled ‘Unskilled and Unaware of It: How Difficulties in Recognizing One’s Own Incompetence Lead to Inflated Self-Assessments’.[1] Their research indicated that people who are unskilled, or lacking academic or professional qualifications in a particular field, have a tendency to estimate their knowledge and skills in that field unrealistically highly.

Citing the observation of Charles Darwin that ‘ignorance more frequently begets confidence than does knowledge’,[2] Dunning and Kruger explained the cognitive process by which people over-estimate their competence in fields concerning which they are unskilled or uninformed. In agreement with the popular saying ‘a little learning is a dangerous thing’,[3] Dunning and Kruger noted ‘in order for the incompetent to overestimate themselves, they must satisfy a minimal threshold of knowledge, theory, or experience that suggests to themselves that they can generate correct answers’.[4]

Accordingly, we ought to refrain from commenting authoritatively on subjects concerning which we are not academically informed or professionally qualified, and should instead seek to understand the subject from the relevant professional literature, instead of from non-professionals and those who are insufficiently qualified.

Additionally, we should be prepared to accept that our non-professional personal views (and the views of others who are similarly unqualified in the field), are of considerably less value than the existing scholarly literature and consensus, and we should be prepared to accept that the consensus is most likely to be correct, even if it contradicts views or sources which we would prefer to believe are more accurate.

The following dot points list indicators of the Dunning-Kruger effect, based on the reasons given by Dunning and Kruger as to why individuals succumb to the effect, and why they ‘fail, through life experience, to learn that they are unskilled’.[5] The more indicators are present in a specific case, the more likely it is that the individual in question is experiencing the effect.

 

  • Skill-boundary transgression: The individual is seeking to operate as an authority or qualified individual, in a field beyond their personal level of academic and professional qualification.[6]

 

  • Self-identified authority: The individual identifies themselves as sufficiently competent to comment authoritatively on the subject. [7]

 

  • Unrecognized competence: The individual’s self-assessed competence is not recognized by those who are academically and professional competent.[8]

 

  • False peers: The individual believes that the favourable commentary of other unskilled and non-professional individuals, indicates they themselves are sufficiently qualified.[9]

 

  • Scrutiny avoidance: The individual fails to submit their work for professional scrutiny (such as in the relevant scholarly literature), for review by those genuinely qualified.[10]

 

  • Pioneer complex: The individual self-identifies as a pioneer unconvering previously unknown or unrecognized facts; a Copernicus or Galileo.[11]

 

  • Conspiracy claims: The individual explains opposition by qualified professionals as a coordinated attempt to suppress truth, in order to defend the existing scholarly consensus.[12]

 

  • Allocentric [13] claims of bias: The individual explains the difference between their views and those of qualified professionals, as the result of inherent bias on the part of the professionals; accusations of bias are directed at anyone other than themselves, and they claim objectivity.


[1] Kruger & Dunning, ‘Unskilled and Unaware of It: How Difficulties in Recognizing One’s Own Incompetence Lead to Inflated Self-Assessments’, Journal of Personality and Social Psychology (77.6.1121-1134), 1999.

[2] Darwin, ‘The Descent of Man, and Selection in Relation to Sex’ volume 1, p. 4 (1871).

[3] Commonly misquoted as ‘A little knowledge is a dangerous thing’, this phrase is a line from English poet Alexander Pope’s poem ‘An Essay on Criticism’ (1709).

[4] Kruger & Dunning, ‘Unskilled and Unaware of It: How Difficulties in Recognizing One’s Own Incompetence Lead to Inflated Self-Assessments’, Journal of Personality and Social Psychology (77.6.1132), 1999.

[5] Ibid., pp. 1131.

[6] ‘Incompetent individuals, compared with their more competent peers, will dramatically overestimate their ability and performance relative to objective criteria.’, ibid., p. 1122; the importance of formal academic and professional qualifications is that they constitute objective criteria by which competency can be assessed, so we should place less trust in those lacking such qualifications.

[7] ‘These findings suggest that unaccomplished individuals do not possess the degree of metacognitive skills necessary for accurate self-assessment that their more accomplished counterparts possess.’, ibid., p. 1122; we cannot rely on those who are not academically and professionally qualified in a particular field, to assess accurately their own authority and competence in that field.

[8] ‘We propose that those with limited knowledge in a domain suffer a dual burden: Not only do they reach mistaken conclusions and make regrettable errors, but their incompetence robs them of the ability to recognize it.’, p. 1132;. it is far more likely that an unqualified non-professional will be wrong in a given field of specialization, than a qualified professional whose competency has been recognized formally by their equally qualified peers

[9] ‘Second, the bungled robbery attempt of McArthur Wheeler not withstanding, some tasks and settings preclude people from receiving self-correcting information that would reveal the suboptimal nature of their decisions (Einhorn, 1982).’, ibid., p. 1131; by keeping themselves predominantly in the intellectual company of those who agree with them, individuals experiencing the Dunning-Kruger effect place themselves in a setting which typically prevents their errors being exposed, instead keeping them in a kind of intellectual echo chamber in which their views are reinforced by being repeated back to them with approval by those unqualified to assess them competently.

[10] ‘One reason is that people seldom receive negative feedback about their skills and abilities from others in everyday life (Blumberg, 1972; Darley & Fazio, 1980; Goffman, 1955; Matlin & Stang, 1978; Tesser & Rosen, 1975)’, ibid., p. 1131; avoidance of scrutiny by professionals enhances this effect, keeping the unqualified away from those who are best able to expose their errors, and preserving their self-delusion that they are correct.

[11] This is a self-delusional identification since neither Copernicus nor Galileo were ‘gifted amateurs’ opposing a body of professionals (both men were professionals, holding formal teaching positions), and Galileo in particular knew that the subject should be decided by professionals astronomers, placing no value whatsoever on the opinions of the unqualified; writing against the papal edict silencing publications on heliocentrism in the preface of his ‘Dialogue’ (1632), Galileo scorned the unqualified amateur: ‘Complaints were to be heard that advisors who were totally unskilled in astronomical observations ought not to clip the wings of reflective intellects by means of rash prohibitions.’,  Galileo, quoted in Næss, ‘Galileo Galilei: When the Earth Stood Still’, p. 131 (2005).

[12 ‘Third, even if people receive negative feedback, they still must come to an accurate understanding of why that failure has occurred. The problem with failure is that it is subject to more attritional ambiguity to success. For success to occur, many things must go right: The person must be skilled, apply effort, and perhaps be a bit lucky. For failure to occur, the lack of any one of these components is sufficient. Because of this, even if people receive feedback that points to a lack of skill, they may attribute it to some other factor (Snyder, Higgins, & Stucky, 1983; Snyder, Shenkel, & Lowry, 1977).’, ibid., p. 1131; when an unqualified non-professional attributes opposition to or dismissal of their theories by qualified professionals as a conspiracy to maintain the intellectual status quo, the Dunning-Kruger effect is very likely responsible: an example is the Science and Public Policy Institute (a non-profit group in the US which opposes the scientific consensus on global warming), ‘People who are not scientists, or even experts on the subjects they write about often write the SPPI reports, and many convey conspiratorial themes. For example, an SPPI publication by Joanne Nova, who describes herself as a “freelance science presenter, writer, & former TV host”, exemplifies not only the ‘Dunning-Kruger’ effect (Dunning et.al. 2003), but also the inactivist movement’s frustration with mainstream climate science and its inflated sense of victimhood.’, Elshof, ‘Can Education Overcome Climate Change Inactivism?’, Journal for Activism in Science and Technology Education (3.1.25), 2011.

[13] Allocentric means ‘focused on others’, or ‘aimed at others’.

h1

What is quote mining?

February 27, 2012

Quote mining is the dishonest practice of collecting quotations selected out of context, to make authors appear to support a position they do not actually hold. It should be avoided completely by anyone attempting to be an honest human being. Sources practicing quote mining lose any claim to credibility.

Searching for evidence of historical atheism, many atheists appeal to quotations which they claim were written by atheists, but which were actually written by theists, deists, or agnostics.[1] [2]  [3] [4]

Hitler[5] and Stalin[6] are misquoted by some atheists in attempts to claim they were devout Christians whose atrocities were motivated by their Christian belief.

Scientists are often misquoted by those who disagree with them; many have been misquoted dishonestly by Christians, in attempts to support their beliefs. Darwin’s comments on the eye,[7] on fossils,[8] transitional forms,[9] and apparent doubts about evolution,[10] [11] [12] are commonly used in this way.

Other commonly misused quotations include statements by Futuyama,[13] [14] Gould,[15] [16] [17] Kemp,[18] and Mark.[19] Unfortunately, pages could be filled with examples of this kind of shameful conduct by Christians.

Quote mining is also found frequently in the popular media, especially on subjects which generate social controversy, such as global warming. The largest national newspaper in Australia (‘The Australian’), typically expresses skepticism of anthropogenic global warming and climate change, and its writers have been accused repeatedly of quote mining.[20] [21] [22]

Other news agencies criticized for the same tactic in their published opposition to the scientific consensus on anthropogenic global warning are Fox News (US), The Daily Telegraph (Australia), and The Telegraph (UK).


[1] A theist believes in a personal god who can be known by revelation (Christians are theists), and a deist believes in an impersonal god who cannot be known by revelation (some Buddhists sects are deists); neither are atheists.

[2] Commonly misquoted deists (believers in an impersonal god), are Epicurus (Greek philosopher), Xenophanes (Greek philosopher), Anaxagoras (Greek philosopher), Euhemerus (Greek philosopher), Marcus Aurelius (Roman emperor and philosopher), Abul ʿAla Al-Maʿarri (Arabian philosopher), Abu Bakr al-Razi  (Arabian philosopher), Ibn al-Rawandi  (Arabian philosopher), Denis Diderot, François-Marie Arouet (Voltaire), Jean-Jacques Rousseau, John Locke, David Hume, Abraham Lincoln, Benjamin Franklin, Thomas Jefferson, James Madison, Ulysses Grant, George Washington (some historians believe he was a highly unorthodox theist), Thomas Paine, and Samuel Clemens (Mark Twain).

[3] Commonly misquoted theists (believers in a personal god), are Al-Ghazali (Muslim), Al-Haytham (Muslim), Thomas Hobbes, Blaise Pascal (Christian of the Jansenist sect), John Adams (American president, and Unitarian Christian), and Annie Besant (Theosophist and mystic).

[4] Commonly misquoted agnostics (undecided as to whether or not any god exists), are Prodicus (Greek philosopher), Protagoras (Greek philosopher), Cicero (Roman orator, politician, and historian), Omar Khayyam (Persian philosopher, poet, mathematician, and astronomer), Robert Ingersoll, Henry Mencken, Ibn Warraq, Charles Templeton, Bill Maher, Karl Popper, David Attenborough, Marie Curie, Albert Einstein, Émile Durkheim, Stephen Jay Gould, Milton Friedman, Carl Sagan, and Bart Ehrman.

[5] Hitler’s religious beliefs are a matter of scholarly debate, but despite his repeated affirmations of Christianity and professions of belief in a personal God in his public writings and speeches, a number of historians and several people who knew him personally believe that his Christianity was a show, and that although religious he was actually anti-Christian; ‘Michael Reissman’s analysis of Hitler’s conception of God suggests that it had much more to do with a providential deity who had chosen him to lead the German people in accordance with ancient principles of leadership than with any Christian concept. As Goebbels commented at Christmas 1939, ‘The Führer is deeply religious but entirely anti-Christian’.’, Bonney, ‘Confronting the Nazi war on Christianity: the Kulturkampf newsletters, 1936-1939’, p. 20 (2009); ‘Hitler’s own religious views underwent significant change in the latter half of the Third Reich. He gave up on the Protestant Church after three failed attempts to achieve unity within its ranks. It is only in the period after this failure that we begin to see some of the anti-Christian remarks for which he is so famous. In October 1937, Hitler commented privately: “I have been freed, after an intense inner struggle, from the still living and childish imaginnings of religion….I now feel as liberated as a foal in the pasture.” Although he did not say so explicitly, the personalistic tone of the comment reveals that this was primarily a reference to his original Catholic faith, not to all religion per se.’, Steigman-Gall, ‘The Holy Reich: Nazi conceptions of Christianity, 1919-1945’, p. 252 (2003); Speer (Hitler’s Minister of Armaments and War Production, and close personal friend), indicated Hitler’s profession of support for Christianity was politically motivated, ‘Hitler usually concluded this historical speculation by remarking: “You see, it’s been our misfortune to have the wrong religion. Why didn’t we have the religion of the Japanese, who regard sacrifice for the Fatherland as the highest good? The Mohammedan religion too would have been much more compatible to us than Christianity. Why did it have to be Christianity with its meekness and flabbiness?’, Speer, ‘Inside the Third Reich’, p. 96 (1970).

[6] ‘When Stalin was a student at the Tiflis Russian Orthodox Seminary, where his mother sent him to become a priest, he becamse a closet atheist. (Many would late note, however, that his works were influenced by a distinctly biblical style.) His atheism remained rooted in some vague idea of a God in nature. Stalin once read a book by Antole France and was particularly impressed by the following line: “If Napoleon had to choose a religion, he would have chosen the adoration of the sun.” Stalin had circled the word “sun” and written in the margin, “Good!”’, Zubok & Pleshakov, ‘Inside the Kremlin’s Cold War: From Stalin to Khruschev’, p. 20 (1996); while historians do not debate that Stalin was an atheist, his atrocities cannot all be attributed simply to him being an atheist, though his anti-religious atrocities (such as his imposition of state atheism by force and his persecution and murder of hundreds of thousands of people because they were religious), certainly can as they were motivated directly by his atheism.

[7]To suppose that the eye, with all its inimitable contrivances for adjusting the focus to different distances, for admitting different amounts of light, and for the correction of spherical and chromatic aberration, could have been formed by natural selection, seems, I freely confess, absurd in the highest possible degree.’, Darwin, ‘The Origin of Species by Means of Natural Selection’, p. 143 (6th ed. 1882.); although this makes Darwin look as if he is saying the eye could not possibly have evolved, Darwin goes on to explain that he believes the evolution of the eye is both possible and likely, ‘Reason tells me, that if numerous gradations from a simple and imperfect eye to one complex and perfect can be shown to exist, each grade being useful to its possessor, as is certainly the case; if further, the eye ever varies and the variations be inherited, as is likewise certainly the case ; and if such variations should be useful to any animal under changing conditions of life, then the difficulty of believing that a perfect and complex eye could be formed by natural selection, though insuperable by our imagination, should not be considered subversive of the theory.’., ibid., pp 143-144.

[8] ‘The case at present must remain inexplicable; and may be truly urged as a valid argument against the views here entertained.’, ibid., p. 287, however, Darwin goes on to provide a possible explanation, ‘To show that it may hereafter receive some explanation, I will give the following hypothesis.’, p. 287; the context shows Darwin was referring to the lack of pre-Cambrian fossils, but since his time multi-cellular fossils pre-dating the Cambrian era have been discovered, such as the Ediacaran biota.

[9] ‘But, as by this theory, innumerable transitional forms must have existed, why do we not find them embedded in countless numbers in the crust of the earth?’, ibid., p. 134; Darwin goes on to answer his own question, ‘It will be more convenient to discuss this question in the chapter on the Imperfection of the Geological Record; and I will here only state that I believe the answer mainly lies in the record being incomparably less perfect than is generally supposed. The crust of the earth is a vast museum; but the natural collections have been imperfectly made, and only at long intervals of time.’, ibid., p. 134.

[10] ‘Long before the reader has arrived at this part of my work, a crowd of difficulties will have occurred to him. Some of them are so serious that to this day I can hardly reflect on them without being in some degree staggered…’ ibid., p. 133; however, Darwin continues, ‘but, to the best of my judgment, the greater number are only apparent, and those that are real are greater not, I think, fatal to my theory.’, ibid., p. 133.

[11] ‘Not one change of species into another is on record… we cannot prove that a single species has been changed.’, commonly mis-attributed to a book entitled ‘My Life & Letters’ (though Darwin never wrote a book by this title), Darwin’s original words appear in a book written by his son, ‘When we descend to details, we can prove that no one species has changed; [i.e. we cannot prove that a single species has changed] nor can we prove that the supposed changes are beneficial, which is the groundwork of the theory.’, (text in square brackets is in the original), Darwin (ed.), ‘The Life and Letters of Charles Darwin, including an autobiographical chapter: edited by his son Francis Darwin’, volume 2, p. 210 (1911); this was speaking only of the evidence available in his own time, and scientists say such proof does exist today.

[12] Darwin repeatedly insisted that he was not saying he had proof of evolution, but did make clear that he believed it was the best explanation of the observable biological record; ‘I am actually weary of telling people that I do not pretend to adduce direct evidence of one species changing into another, but that I believe that this view in the main is correct, because so many phenomena can be thus grouped together and explained.  But it is generally of no use; I cannot make persons see this.’, Darwin, letter to FW Hutton 20 April, 1861, in Darwin & Seward (eds.), ‘More Letters of Charles Darwin’, letter 124, volume 1, p. 184 (1903).

[13]Undeniably, the fossil record has provided disappointingly few gradual series. The origins of many groups are still not documented at all. Futuyma, ‘Science on Trial: The Case for Evolution’, pp. 190-191 (1983); the title of the book shows Futuyama supported evolution, and the paragraph before this sentence says ‘Contrary to Creationist claims, the transitions among vertebrate species are almost all documented to a greater or lesser extent.’, and the paragraph after the quoted sentence says ‘But in view of the rapid pace evolution can take, and the extreme incompleteness of fossil deposits, we are fortunate to have as many transitions as we do. The creationist argument that if evolution were true we should have an abundance of intermediate fossils is built by denying the richness of paleontological collections, by denying the transitional series that exist, and by distorting, or misunderstanding, the genetical theory of evolution.’, ibid., pp. 190, 191.

[14] ‘The majority of major groups appear suddenly in the rocks, with virtually no evidence of transition from their ancestors.’, Futuyma, ‘Science on Trial: The Case for Evolution’, p. 82 (1983); on the next page Futuyma writes ‘The transitional forms that evolve so quickly, and in such a small area, are very unlikely to be picked up in the fossil record. Only when the newly evolved species extends its range will it suddenly appear in the fossil record. Eldredge and Gould have suggested, therefore, that the fossil record should show stasis, or equilibrium, of established species, punctuated occasionally by the appearance of new forms. Hence, the fossil record would be most inadequate exactly where we need it most — at the origin of major new groups of organisms.’, ibid., p. 83.

[15] ‘Paleontologists have paid an enormous price for Darwin’s argument. We fancy ourselves as the only true students of life’s history, yet to preserve our favored account of evolution by natural selection we view our data as so bad that we almost never see the very process we profess to study.’, Gould, ‘The Panda’s Thumb: More Reflections in Natural History’, p. 181 (1980); the next sentence says ‘We believe that Huxley was right in his warning. The modern theory of evolution does not require gradual change. In fact, the operation of Darwinian processes should yield exactly what we see in the fossil record. It is gradualism we should reject, not Darwinism.’, ibid., p. 181.

[16] ‘The fossil record had caused Darwin more grief than joy. Nothing distressed him more than the Cambrian explosion, the coincident appearance of almost all complex organic designs…’, ibid. pp. 238-239; Gould later writes ‘His opponents interpreted this event as the moment of creation, for not a single trace of Precambrian life had been discovered when Darwin wrote the Origin of Species. (We now have an extensive record of monerans from these early rocks, see essay 21).’, ibid., p. 239.

[17] ‘We have long known about stasis and abrupt appearance, but have chosen to fob it off upon an imperfect fossil record.’, Gould, ‘The Paradox of the First Tier: An Agenda for Paleobiology’, Paleobiology, (11.1.7), 1985; this is a corruption of what Gould actually wrote, which was ‘Just as we have long known about stasis and abrupt appearance, but have chosen to fob it off up on an imperfect fossil record, so too have we long recognized the rapid, if not sudden, turnover of faunas in episodes of mass extinction.’, ibid., p. 7.

[18] ‘In other words, when the assumed evolutionary processes did not match the pattern of fossils that they were supposed to have generated, the pattern was judged to be ‘wrong.’ A circular argument arises: interpret the fossil record in terms of a particular theory of evolution, inspect the interpretation, and note that it confirms the theory. Well, it would, wouldn’t it? …As is now well known, most fossil species appear instantaneously in the record, persist for some millions of years virtually unchanged, only to disappear abruptly – the ‘punctuated equilibrium’ pattern of Eldredge and Gould.’, Kemp, ‘A Fresh Look at the Fossil Record’, New Scientist  (108. 66), 1985; Kemp was criticizing an approach used by some earlier scientists (but abandoned by his time), not denying evolution or claiming that methods used by scientists are wrong, and his next sentence makes this clear, ‘Irrespective of one’s view of the biological causes of such a pattern (and there continues to be much debate about this), it leads in practice to description of long-term evolution, or macroevolution, in terms of the differential survival, extinction and proliferation of species. The species is the unit of evolution.’, ibid., p. 66.

[19] ‘In any case, no real evolutionist, whether gradualist or punctuationist, uses the fossil record as evidence in favour of the theory of evolution as opposed to special creation.” Mark, ‘Who doubts evolution?’ New Scientist, (90.831) 1981; statements from the article making it clear that Mark was actually arguing against special creation and pointing out that scientists use several lines of evidence to prove evolution (not simply the fossil records, which is the least important), include ‘Someone is getting it wrong, and it isn’t Darwin; it is the creationists and the media.’ (p. 830), ‘So what is the evidence that species have evolved? There have traditionally been three kinds of evidence, and it is these, not the “fossil evidence”, that the critics should be thinking about. The three arguments are from the observed evolution of species, from biogeography, and from the hierarchical structure of taxonomy.’ (p. 831), ‘These three are the clearest arguments for the mutability of species. Other defences of the theory of evolution could be made, not the least of which is the absence of a coherent alternative. Darwin’s theory is also uniquely able to account for both the presence of design, and the absence of design (vestigial organs), in nature.’ (p. 832).

[20] ‘On climate change, The Australian is behaving like the media equivalent of a fog machine. Its unreliable reporting should be avoided by those with an interest in factual scientific information.’, Jones, ‘Spinning uncertainty? The IPCC extreme weather report and the media’, The Conversation, 23 November 2011.

[21]The Australian has a daily column called Cut and Paste which should more properly be titled Quote Mining.’, Lambert, ‘The Australian’s War on Science 58: Quote Mining’, Deltoid, 4 February 2011.

[22] ‘Unfortunately for Kerr, [writer for The Australian] the report is available online, so we can see how Kerr quote mined it:.’, Lambert, ‘The Australian’s War on Science 63: Quote Mining’, Deltoid, 14 July 2011.

h1

Is the Bible’s chronology of the kings of Israel accurate?

July 30, 2011

The Challenge

In the late 19th century, critical scholar Julius Wellhausen claimed the Biblical chronology of the kings of Israel was a literary invention for religious purposes, which had been edited and revised several times from a variety of different sources, rather than a genuine historical record.[1]

For the next 70 years, critical scholars continued to treat the chronology as historically worthless and irreconcilable.[2]

The Facts

In 1951, Biblical scholar Edwin Thiele published ‘The Mysterious Numbers of the Hebrew Kings’, a harmonization of the Biblical record of the kings of Israel (originally as a doctoral dissertation). By the time of the second edition (slightly revised), it was recognized that Thiele’s work was a significant breakthrough in establishing the historical validity of the Biblical chronology.[3]

Reception

Though criticisms have been made of Thiele’s chronology,[4] [5] [6] [7] its value and general validity have been acknowledged widely.[8] [9] [10] [11] [12]

It remains the typical starting point for study of the chronology of the kings of Israel and Judah[13] [14] with few modifications,[15] [16]   and has been applied successfully in other fields of Ancient Near East study, such as the chronologies of Assyria and Babylon.[17]

The reliability of the chronologies in 1-2 Kings has been supported by archaeological evidence; Grabbe notes that the chronology in these books ‘agrees with what can be gleaned from extra-biblical sources’, and that ‘even if we had no external sources we could have reasonable confidence in the biblical sequence’.[18]


[1] ‘That a process of alteration and improvement of the chronology was busily carried on in later times, we see from the added synchronisms of the kings of Israel and Judah,’, Wellhausen, ‘Prolegomena to the History of Israel’, p. 278 (1885).

[2] ‘Driver remarked that, “the length of the reigns of the various kings is not the same according to the traditional and the synchronistic figures. Since, however, it is clear on various grounds that these synchronisms are not original, any attempt to base a chronological scheme on them may be disregarded.” Kittel stated his view that, “Wellhausen has shown, by convincing reasons, that the synchronisms within the Book of Kings cannot possibly rest on ancient tradition, but are on the contrary simply the products of artificial reckoning. . . The Israelitish numbers and the parallel numbers referring to Judah do not agree at the points at which we are able to compare them.” Robinson also was impressed by Wellhausen’s evaluation: “Wellhausen is surely right in believing that the synchronisms in Kings are worthless, being merely a late compilation from the actual figures given.” * R. H. Pfeiffer’s opinion was that, “The chronology based on the synchronisms is of course less reliable than the one based on the regnal periods, since the synchronisms were figured from the regnal periods. Neither chronology is wholly accurate . . . In spite of these discrepancies, inaccuracies, and errors, the chronology of Kings is not fantastic.” 5 J. F. McCurdy expressed himself to the effect that, “Many of the numbers given, especially the synchronisms, are erroneous, as is proved by the fact that no attempt to harmonize the two series has been successful . . . Startling inconsistencies are also found where the several synchronisms for the same king are worked out.” K. Marti gave his observation: “The synchronistic notes betray their character as ‘subjective additions of the Epitomator.’ It is clear, to begin with, that this noting of synchronisms was not in actual use during the existence of the two kingdoms. . . Almost along the whole line, the discrepancy between synchronisms and years of reign is incurable.” C. H. Gordon observed: “The numerical errors in the Books of Kings have defied every attempt to ungarble them. Those errors are largely the creation of the editors who set out to write a synchronistic history of Judah and Israel, using as sources two sets of unrelated court chronicles. Combining two elaborate sets of figures was not an easy task. But even with due regard for the difficulties involved, the editors did not execute the synchronisms skillfully.”’, Thiele, ‘Synchronisms of the Hebrew Kings – A Re-evaluation: I’, Andrews University Seminary Studies, pp. 14-125 (1), 1963.

[3]A marked advance in biblical scholarship was made in the publication of The Mysterious Numbers of the Hebrew Kings, U. of Chicago Press, 1951, by Dr. E. R. Thiele. In his revised edition in 1965 (Eerdmans Pub. Co., Grand Rapids, Mich.), Dr. Thiele asserts the soundness of his basic thesis and conclusions as confirmed by scholars since his first edition.’, editorial, Journal of the Evangelical Theological Society (9.1.60), 1966.

[4] ‘Thiele’s view contains positive elements, but it also poses numerous difficulties. He incorrectly understood the annals of Tiglath-pileser III, and his determination that Menahem died in 742 contradicts the testimonies of the contemporaneous Assyrian inscriptions.10 In his desire to resolve the discrepancies between the data in the Book of Kings, Thiele was forced to make improbable suppositions. He assumed that the system of counting the years of reign changed every few generations, or even after a few decades. This is improbable, and cannot be proved. Similarly, he presumes that the Northern and Southern Kingdoms numbered their years both by the local count and by that practiced by the other kingdom, also for short periods, while this practice ceased in other periods. Thiele even went so far as to assume that while this practice had fallen into disuse, there were scribes who continued to calculate the years in accordance with it. There is no basis for Thiele’s statement that his conjectures are correct because he succeeded in reconciling most of the data in the Book of Kings, since his assumptions regarding Biblical chronological principles are derived from the chronological data themselves, whose reliability is unclear.’, Galil, ‘The Chronology of the Kings of Israel and Judah’, p. 4 (1996).

[5]but his harmonizing approach has not gone unchallenged, especially because of the many shifts in the basis of reckoning dates that it requires (e.g., Jepsen 1968: 34–35)—shifts which were unlikely in actual practice. The numerous extrabiblical synchronisms he invokes do not always reflect the latest refinements in Assyriological research (cf. E.2.f below). In many cases, he posits an undocumented event in order to save a biblical datum (e.g., the circumstances surrounding the appointment of Jeroboam II as coregent; Thiele 1983: 109)’, Cogan, ‘Chronology (Hebrew Bible)’, in Freedman, (ed.), ’The Anchor Yale Bible Dictionary’, volume 1, p. 1006 (1996)’, Freedman, DN (1996).

[6]  ‘Despite that fact of scholarly dedication, neither Thiele’s carefully argued University of Chicago dissertation, nor anyone else’s, has achieved as yet universal acceptance.’, Kaiser, ‘A History of Israel: From the bronze age through the Jewish Wars’, p. 293 (1998).

[7]Not all scholars are convinced by this solution, and commentators on the prophetic books often accept that dates can only be approximate.’, McConville, ‘Exploring the Old Testament, Volume 4: The Prophets’, p. viii (2002).

[8] ‘The chronology most widely accepted today is one based on the meticulous study by Thiele. Wiseman, ‘1 and 2 Kings’, Tyndale Old Testament Commentaries, p. 27 (1993).

[9]Increasingly his chronological scheme has come to dominate the majority of scholarly works and it is unlikely that his system can ever be overthrown without altering some well-established dates in Near Eastern history, for Thiele’s chronology is now inextricably locked into the chronology of the Near East.’, McFall, ‘A Translation Guide to the Chronological Data in Kings and Chronicles’, Bibliotheca Sacra (148.589.42-43), 1996.

[10] ‘Thiele’s system of chronology has been well received over the past 40 years and is now accepted as the basis for Israel’s chronology in a growing number of standard scholarly works.’, ibid., p. 42; see for example: Mitchell, ‘Israel and Judah until the Revolt of Jehu (931-841 B.C.)’, Cambridge Ancient History, volume 3, part 1, p. 445 (1982); Finegan, ‘Handbook of Biblical Chronology’, p. 249 (rev. ed.1998); Hess, ‘Chronology (Old Testament)’, in Porter (ed.), ‘Dictionary of Biblical Criticism and Interpretation’, p. 55 (2007).

[11]Thiele’s chronology is fast becoming the consensus view among Old Testament scholars, if it has not already reached that point.’, McFall, ‘The Chronology of Saul and David’, Journal of the Evangelical Theological Society (53.101.215), 2010.

[12]Thiele’s chronology (which differs from that of the present paper in only a few places) won the respect of historians because its dates agree with the following dates in Assyrian and Babylonian history: the Battle of Qarqar in 853 bc; the tribute of Jehu to Shalmaneser III in 841 bc; the capture of Samaria by Shalmaneser V in 723 bc; the invasion of Sennacherib in 701 bc; the Battle of Carchemish in 605 bc; and the first capture of Jerusalem by Nebuchadnezzar in 597 bc.’, Young, ‘Tables of Reign Lengths From the Hebrew Court Recorders’, Journal of the Evangelical Theological Society (48.2.232), 2005.

[13] ‘Thiele’s work has become a cornerstone of much recent chronological discussion (cf. De Vries IDB 1: 580–99; IDBSup: 161–66);’, Cogan, ‘Chronology (Hebrew Bible)’, in Freedman, (ed.), ’The Anchor Yale Bible Dictionary’, volume 1, p. 1006 (1996)’, Freedman, DN (1996).

[14] ‘Although some would prefer to see transmission errors where Thiele invokes the above principles, his chronology remains the starting point for all discussions of the debate.’, Hess, ‘Chronology (Old Testament)’, in Porter (ed.), ‘Dictionary of Biblical Criticism and Interpretation’, p. 55 (2007).

[15] ‘After 40 years Thiele’s chronology has not been significantly altered or proved to be false in any major area except in the matter of Hezekiah’s coregency.’, McFall, ‘A Translation Guide to the Chronological Data in Kings and Chronicles’, Bibliotheca Sacra (148.589.42), 1996.

[16]It remained then for others to complete the application of principles that Thiele used elsewhere, thereby providing a chronology for the eighth-century kings of Judah that is in complete harmony with the reign lengths and synchronisms given in 2 Kings and 2 Chronicles. The most thorough work in this regard was Leslie McFall’s 1991 article in Bibliotheca Sacra.22 McFall made his way through the reign lengths and synchronisms of Kings and Chronicles, and using an exact notation that indicated whether the years were being measured according to Judah’s Tishri years or Israel’s Nisan years, he was able to produce a chronology for the divided monarchies that was consistent with all the scriptural texts chosen.’, ibid., pp. 105-106.

[17] ‘In a 1996 article, Kenneth Strand wrote, “What has generally not been given due notice is the effect that Thiele’s clarification of the Hebrew chronology of this period of history has had in furnishing a corrective for various dates in ancient Assyrian and Babylonian history.”28 The purpose of Strand’s article was to show that Thiele’s methodology accomplished more than just producing a coherent chronology from scriptural data. His chronology, once produced, proved useful in settling some troublesome problems in Assyrian and Babylonian history.’, Young, ‘Inductive And Deductive Methods As Applied To OT Chronology’, Master’s Seminary Journal (18.1.112-113), 2007.

[18] ‘Grabbe suggests that the names and sequence of kings in Israel and Judah, and their approximate chronological placement, agrees with what can be gleaned from extra-biblical sources. To this extent the biblical framework (meaning primarily 1 and 2 Kings) is reliable: even if we had no external sources we could have reasonable confidence in the biblical sequence of Jeroboam I, Nadab, Baasha, Elah, Omri, Ahab, Jehu, etc. in Samaria, and David, Solomon, Rehoboam, Abijam, Asa, Jehoshaphat, etc. in Jerusalem, along with their interrelationships. Beyond that it starts to get more and more tricky, with decreasing reliability in the biblical narrative as the detail increases (this is a general statement, and there are sometimes exceptions in specific instances).’, Grabbe, ‘Reflections on the Discussion’, Grabbe (ed.), ‘Ahab Agonistes: The Rise and Fall of the Omri Dynasty’, p. 337 (2007).

h1

What benefits are there to being religious?

June 21, 2011

The Challenge

Atheist authors Christopher Hitchens[1] and Richard Dawkins[2] have condemned religious belief as harmful and useless. Both authors have been criticized for failing to note specific benefits of strong religious belief. [3] [4] [5] [6] [7]

The Facts

Strong religious belief provides a range of physical, emotional, social, financial, psychological, and health benefits;[8] [9] reducing health risks, increasing the likelihood of longevity[10] and mental health,[11] and having a positive effect on wellbeing in childhood,[12] [13] [14] as well as later economic outcomes.[15]

High religious involvement has a positive effect on social integration,[16] [17] [18] behavioural regulation,[19] and a range of positive societal outcomes.[20] However, lower levels of religious belief, involvement, and commitment produce negative outcomes.[21] [22]


[1] ‘God Is Not Great: How Religion Poisons Everything’ (2007).

[2] ‘The God Delusion’ (2006).

[3] ‘Unfortunately, however, both authors either fail to appreciate, or have chosen not to acknowledge, the extraordinary importance that a very “personal God” may play in the lives of many individuals forced to deal with these profoundly life-changing situations.’, Markman, ‘Benefits of Religious Beliefs for Cancer Patients: A Response to Dawkins and Hitchens’, Current Oncology Reports (10.185), 2008.

[4] ‘But there is increasingly strong evidence that when confronted with a life-changing challenge such as being diagnosed with a malignancy, a genuine human need exists, as noted by Professor Dawkins and Mr. Hitchins’s headmaster, for the presence of personal spiritual support.’, ibid., p. 185.

[5] ‘Empiric evidence exists that a cancer patient’s ability to successfully deal with spiritual issues at the end of life is associated with less overwhelming despair and intense feelings of hopelessness [15]. Effective coping with these concerns can favorably impact the quality of life [16]. Existing data also support the hypothesis that it is the impact of a general feeling of spiritual well-being—not specific religious beliefs or practices—that is correlated with the favorable effect [17].’, ibid., p. 186.

[6]the overwhelming existing evidence demonstrates that some patients with malignant disease may experience considerable benefit from a strong sense of spiritual well being and the presence of a “personal God.”’, ibid., p. 187.

[7] ‘It is unfortunate that Professor Dawkins and Mr. Hitchens were not willing to fully acknowledge the relevance of these points.’, ibid., p. 187.

[8]Many studies have documented the benefits of religious involvement. Indeed, highly religious people tend to be healthier, live longer, and have higher levels of subjective well-being.’, Mochon, Norton, & Ariely, ‘Who Benefits from Religion?’, Social Indicators Research (101.1), 2010.

[9] It is recognized that not all religious systems provide such benefits; for example, some groups result in negative outcomes due to prejudice against education, or to oppressive power structures and failure to reinforce positive behaviours.

[10] ‘Similarly, although there are exceptions and the matter remains controversial (Sloan et al. 1999), a growing body of research documents an association between religious involvement and better outcomes on a variety of physical health measures, including problems related to heart disease, stroke, hypertension, cancer, gastrointestinal disease, as well as overall health

status and life expectancy. This research also points to differences by religious affiliation, with members of stricter denominations displaying an advantage (Levin 1994). Many of the early studies in this literature suffer from methodological shortcomings, including small, unrepresentative samples, lack of adequate statistical controls, and a cross-sectional design that confounds the direction of causality. Yet the conclusion of a generally positive effect of religious involvement on physical health and longevity also emerges from a new generation of studies that have addressed many of these methodological problems (Ellison and Levin 1998). In one of the most rigorous analyses to date, Hummer et al. (1999) use longitudinal data from a nationwide survey, the 1987 Cancer Risk Factor Supplement–Epidemiology Study, linked to the Multiple Cause of Death file. Their results show that the gap in life expectancy at age 20 between those who attend religious services more than once a week and those who never attend is more than seven years—comparable to the male–female and white–black differentials in the United States. Additional multivariate analyses of these data reveal a strong association between religious participation and the risk of death, holding constant socioeconomic and demographic variables, as well as initial health status. Other recent longitudinal studies also report a protective effect of religious involvement against disability among the elderly (Idler and Kasl 1992), as well as a positive influence on self-rated health (Musick 1996) and longevity (Strawbridge et al. 1997).’, Waite & Lehrer, ‘The Benefits from Marriage and Religion in the United States: A Comparative Analysis’, p. 2 (author manuscript 2003).

[11] ‘The connection between religion and mental health has been the subject of much controversy over the years, and many psychologists and psychiatrists remain skeptical, in part because most of the research has been based on cross-sectional analyses of small samples. The studies to date are suggestive of an association between religious involvement and better mental health outcomes, including greater self-esteem, better adaptation to bereavement, a lower incidence of depression and anxiety, a lower likelihood of alcohol and drug abuse, and greater life satisfaction and happiness in general (Koenig et al. 2001). Recent longitudinal analyses of subgroups of the population provide additional evidence in support of this relationship (Zuckerman et al. 1984; Levin et al. 1996).’, ibid., p. 3.

[12] Religious participation has also been associated with better educational outcomes. Freeman (1986) finds a positive effect of churchgoing on school attendance in a sample of inner-city black youth. Regnerus (2000) reports that participation in religious activities is related to better test scores and heightened educational expectations among tenth-grade public school students. In the most comprehensive study to date, using data on adolescents from the National Education Longitudinal Study of 1988, Muller and Ellison (2001) find positive effects of various measures of religious involvement on the students’ locus of control (a measure of self-concept), educational expectations, time spent on homework, advanced mathematics credits earned, and the probability of obtaining a high school diploma.’, ibid., p. 4.

[13] ‘Several studies have documented an association between religion and children’s well-being. Recent research on differences in parenting styles by religious affiliation reveals that conservative Protestants display distinctive patterns: they place a greater emphasis on obedience and tend to view corporal punishment as an acceptable form of child discipline; at the same time, they are more likely to avoid yelling at children and are more prone to frequent praising and warm displays of affection (Bartowski et al. 2000). As to other dimensions of religion, Pearce and Axinn (1998) find that family religious involvement promotes stronger ties among family members and has a positive impact on mothers’ and children’s reports of the quality of their relationship. A number of studies document the effects of children’s own religious participation, showing that young people who grow up having some religious involvement tend to display better outcomes in a range of areas. Such involvement has been linked to a lower probability of substance abuse and juvenile delinquency (Donahue and Benson 1995), a lower incidence of depression among some groups (Harker 2001), delayed sexual debut (Bearman and Bruckner 2001), more positive attitudes toward marriage and having children, and more negative attitudes toward unmarried sex and premarital childbearing (Marchena and Waite 2001).’, ibid., p. 4.

[14] ‘Overall, we find strong evidence that youth with religiously active parents are less affected later in life by childhood disadvantage than youth whose parents did not frequently attend religious services. These buffering effects of religious organizations are most pronounced when outcomes are measured by high school graduation or non-smoking and when disadvantage is measured by family resources or maternal education, but we also find buffering effects for a number of other outcome-disadvantage pairs. We generally find much weaker buffering effects for other social organizations.’, Dehejia et al., ‘The Role of Religious and Social Organizations in the Lives of Disadvantaged Youth’, NBER Working Paper No. 13369 (2007).

[15] ‘However, as we discuss below, an emerging literature shows a positive effect of religiosity on educational attainment, a key determinant of success in the labor market. These studies suggest a potentially important link between religious involvement during childhood and adolescence and subsequent economic well-being as an adult. Preliminary results from a new line of inquiry at the macro level are consistent with this hypothesis. Using a cross-country panel that includes information on religious and economic variables, Barro and McLeary (2002) find that enhanced religious beliefs affect economic growth positively, although growth responds negatively to increased church attendance. The authors interpret their findings as reflecting a positive association between “productivity” in the religion sector and macroeconomic performance.’, ibid., p. 3.

[16] ‘Ellison and George (1994) find that people who frequently attend religious services not only have larger social networks, but also hold more positive perceptions of the quality of their social relationships.’, Waite & Lehrer, ‘The Benefits from Marriage and Religion in the United States: A Comparative Analysis’, p. 7 (author manuscript 2003).

[17] ‘Recent research has emphasized that religion can play a pivotal role in the socialization of youth by contributing to the development of social capital. Religious congregations often sponsor family activities, stimulating the cultivation of closer parent–child relations; they also bring children together with grandparents and other supportive adults (parents of peers, Sunday-school teachers) in an environment of trust. This broad base of social ties can be a rich source of positive role models, confidants, useful information, and reinforcement of values that promote educational achievement.’, ibid., p. 7.

[18] ‘At the other end of the age spectrum, the social ties provided by religious institutions are of special value to the elderly, helping them deal with the many difficult challenges that tend to accompany old age: illness, dependency, loss, and loneliness (Levin 1994).’, ibid., p. 7.

[19]Most faiths have teachings that encourage healthy behaviors and discourage conduct that is self-destructive; they also provide moral guidance about sexuality. Some religions have specific regulations limiting or prohibiting the consumption of alcohol, tobacco, caffeine, and potentially harmful foods. Several studies show that religious involvement is generally associated with health-promoting behaviors (Koenig et al. 2001) and that such behaviors explain in part the connection between religion and longevity (Strawbridge et al. 1997; Hummer et al. 1999).’, ibid., p. 7.

[20] ‘At the societal level, higher religious involvement is related to increased levels of education (Gruber 2005), lower crime rates (Baier and Wright 2001; Johnson et al. 2000), increases in civic involvement (Putnam 2000; Ruiter and De Graaf 2006), higher levels of cooperation (Norenzayan and Shariff 2008; Shariff and Norenzayan 2007), lower divorce rates, higher marital satisfaction and better child adjustment (Mahoney et al. 2001; for a review, see Sherkat and Ellison 1999).’, Mochon, Norton, & Ariely, ‘Who Benefits from Religion?’, Social Indicators Research (101.2), 2010.

[21]While fervent believers benefit from their involvement, those with weaker beliefs are actually less happy than those who do not ascribe to any religion—atheists and agnostics..’, ibid., p. 1.

[22]Indeed, weakly affiliated adherents may actually be less happy than their unaffiliated counterparts—atheists, agnostics, and those who report no religion at all—and therefore would appear to benefit from abandoning their faith.’, ibid., p. 2.

h1

Was the Genesis flood narrative copied from Mesopotamian myths?

June 1, 2011

The Challenge

By the end of the 19th century archaeology had discovered many Mesopotamian texts containing creation and flood narratives remarkably similar to those in the Bible.

Critical scholars came to believe that the Biblical narratives had simply been copied from earlier Mesopotamian myths.[1] [2]The Biblical flood narrative in particular is still considered by some scholars to have been borrowed from the Mesopotamian story.[3] [4]

The Facts

Later scholarship noted significant differences between the Biblical and Mesopotamian narratives;[5] the Mesopotamian creation narratives were now viewed as parallels to the Genesis narrative.[6]  Still later it was the Genesis and Babylonian accounts shared an earlier Mesopotamian source, whether literary or oral.[7] [8] [9]

Scholarly Views

Kitchen (Assyriologist), note that Assyrologists have abandoned the idea of Genesis 1-11 being borrowed from Assyrian and Babylonian texts.[10] Millard (Professor Emeritus of Hebrew and Ancient Semitic Languages), observes there is no evidence for direct literary borrowing.[11] [12] This is the majority view of current scholarship. [13]  [14] [15] [16] [17] [18] [19] [20]

At least early as 1872, it was suggested that the similarities between the Genesis and Mesopotamian flood narratives are due to the texts describing the same genuine historical event. [21]

In the early 20th century, critical scholar Hermann Gunkel observed that this was supported by the curious description (in both the Genesis account and the earlier Mesopotamian accounts), of the Ark being driven upstream, contrary to expectation.[22]  This explanation remains well represented in scholarship. [23] [24] [25] [26] [27]


[1] ‘Some argued that many Hebrew ideas actually originated in Mesopotamia and were borrowed by Israel.’, Chavalas, ‘Mesopotamia and the Bible’, p. 32 (2003).

[2] ‘The idea of Babylonian primacy was perfected by Delitzsch in 1902-1903. In his lectures, he argued that Israel could only be studied in light of Babylonia, and in fact Israelite civilization was derived from Babylonia.’, ibid., p. 32.

[3] ‘Since this portion of the biblical narrative postdates the Mesopotamian traditions (the final form of this portion of Genesis is usually dated to the fifth century B.C.E, although its oral or written sources may be dated as much as six hundred years earlier), it is conceivable, if not likely, that the biblical writer has borrowed and adapted Mesopotamian flood traditions.’, Fant & Reddish, ‘Lost Treasures of the Bible: Understanding the Bible through Archaeological Artifacts in World Museums’, p. 25 (2008).

[4] ‘It is commonly accepted that parts of Genesis 1–11 show literary dependence, either directly or indirectly, on Mesopotamian literary tradition.187 The best test case would be the flood story in Genesis 69.’, Smith, ‘God in Translation: Deities in Cross-Cultural Discourse in the Biblical World’, p. 182 (2010).

[5] ‘As scholars studied the significant differences and omissions between the accounts, they concluded that neither the Mesopotamian nor the biblical author borrowed from the other.’, Couch, ‘The Fundamentals for the Twenty-First Century: Examining the Crucial Issues of the Christian Faith’, p. 177 (2000).

[6] ‘Nevertheless, it adds much that is significant for the Near Eastern mythological horizon, and perhaps even provides a number of interesting parallels to the motifs of the biblical paradise story as told in the second and third chapters of Genesis.’, Kramer, ‘Sumerian Myths and Epic Tales’, in Pritchard (ed.), ‘Ancient Near Eastern texts relating to the Old Testament’, p. 37 (1950).

[7] ‘The similarities in broad outline and in certain points of detail between the Gilgamesh and the Genesis and the Gilgamesh versions are too striking to be accidental. Both probably derive from a common older Mesopotamian tradition, fragments of which are preserved in the Sumerian version.’, Davidson, ‘Genesis 1-11’, Cambridge Bible Commentaries p. 65 (1973).

[8] ‘It is undoubtedly borrowed from a common religious tradition of flood accounts.’, Brueggemnann, ‘Genesis’, p. 73 (1982).

[9] ‘Although the differences between the two stories may be too great to support a theory of direct literary dependence, most scholars are convinced that the biblical flood narrative is to some degree dependent upon ancient Mesopotamian flood narratives.’, Fant & Reddish, ‘Lost treasures of the Bible: understanding the Bible through archaeological’, p. 21 (2008).

[10]Thus most Assyriologists have long since rejected the idea of any direct link between Gen. 1-11 and Enuma Elish, and nothing else better can be found between Gen. 1-11 and any other Mesopotamian fragments.’, Kitchen, ‘On the Reliability of the Old Testament’, p. 424 (2003); his footnote reads ‘Assyriologists generally reject any genetic relationship between Gen. 1-2 and the Mesopotamian data because of the considerable differences; see (eg.) J.V. Kinnier-Wilson. In D. W. Thomas, ed., Documents from Old Testament Times (London: Nelson, 1958), 14; W. G. Lambert, JTS. n.s., 16 (1965): 287-300, esp. 289. 291, 293-99. and in ISF, 96-113, with addenda; A. R. Millard, TynB 18 (1967): 3-4.7. 16-18, and in ISIF 114-28; T. Jacobsen, in JBL 100 (198 1): 513-29, and translation, both now in ISIF 129-42, plus 160-66.’, ibid., p. 591.

[11]However, it has yet to be shown that there was borrowing, even indirectly. Differences between the Babylonian and the Hebrew traditions can be found in factual details of the Flood narrative (form of the Ark; duration of the Flood, the identity of the birds and their dispatch) and are most obvious in the ethical and religious concepts of the whole of each composition. All who suspect or suggest borrowing by the Hebrews are compelled to admit large-scale revision, alteration, and reinterpretation in a fashion that cannot be substantiated for any other composition from the ancient Near East or in any other Hebrew writing. If there was borrowing then it can have extended only as far as the “historical” framework, and not included intention or interpretation.’, Millard, ‘A New Babylonian “Genesis” Story’, in  Hess & Tsumura (eds.), ‘I Studied Inscriptions from Before the Flood: Ancient Near Eastern, Literary Approaches to Genesis 1-11’, Sources for Biblical and Theological Study, volume 4, p. 127 (1994).

[12] ‘The two accounts undoubtedly describe the same Flood, the two schemes relate the same sequence of events. If judgment is to be passed as to the priority of one tradition over the other, Genesis inevitably wins for its probability in terms of meteorology, geophysics, and timing alone.’, ibid., pp. 127-128.

[13] ‘The similarities between the Genesis account and the ‘Atra-Hasis Epic’ do not support the idea that Genesis is a direct borrowing from the Mesopotamian but do indicate that Mesopotamian materials could have served as models for Genesis 1-11, as Jacobsen holds. P.D. Miller also admits that ‘there were Mesopotamian models that anticipate the structure of Genesis 1-11 as a whole.’, Tsumura, ‘Genesis and Ancient Near Eastern Stories of Creation and Flood’, in ibid., p. 47.

[14] ‘With Genesis 1-11 we seem to be working more with shared motifs and basic plotlines that originated in Mesopotamia rather than with actually known texts directed [sic] borrowed into Israel.’, Smith, ‘God in Translation: Deities in Cross-Cultural Discourse in the Biblical World’, p. 182 (2010).

[15] ‘The Bible’s accounts of the creation of the world, the creation of humankind, and the flood were not borrowed from these, but neither are they unique in every respect.’, Arnold & Beyer (eds.), ‘Readings from the ancient Near East: primary sources for Old Testament study’, p. 13 (2002).

[16] ‘The details are not exact and most scholars deny any direct literary dependence but it would seem that both stories emerge from a common tradition or milieu.’, Moyise, ‘Introduction to Biblical Studies’, p. 33 (2004).

[17] ‘The Biblical flood of Noah in the book of Genesis 6-9 shares continuity with the other Ancient Near Eastern flood stories, but is probably not directly dependent on any of them.’, Snell, ‘A Companion to the Ancient Near East’, p. 256 (2005).

[18] ‘But after a careful study of the two, Alexander Heidel has concluded that “no incontrovertible evidence can for the present be produced” in favor of biblical dependence on the Babylonian materials. His conclusion regarding the flood accounts is similar.’, Niehaus, ‘Ancient Near Eastern Themes in Biblical Theology’, p. 22 (2008).

[19] Text

[20] ‘Many who have done thorough linguistic and literary analysis (e.g., A. Heidel, A.R. Millard, D. Damrosch) conclude that literary dependence cannot be demonstrated. Here, as in most of the parallels in the primeval history, it is considered more likely that Mesopotamian and biblical traditions are based on a common source. Some understand this common source to be a piece of more ancient literature, while others consider it the actual event.’, Hill & Walton, ‘A Survey of the Old Testament’, p. (2010).

[21] ‘Among many theorists, George Smith in 1872 [33] famously linked the great Biblical Flood of the book Genesis to an historical event, probably of the 3rd millennium BC, which deposited a 50-cm- sediment-layer in the Mesopotamian lowland.’, Haigh & Křeček, ‘Environmental Reconstruction in Headwater Areas’, p. 14 (2000).

[22] ‘The most characteristic element of the Babylonian account seems to be that the Ark, driven from the South inland against the current of the rivers, was stranded in the northern mountains. This element is so remarkable that it could only have been stimulated by a corresponding natural phenomenon. E. Suss (25ff.) suspects that a violent earthquake in the Persian Gulf may have been the cause. A powerful cyclone from the South, associated with voluminous rain and horrible darkness, drove the destructive waters far into the inhabited land. This event must have taken place in a very ancient time. The news of the terrible catastrophe was preserved through all times. This theory is certainly very plausible.’, Gunkel ‘Genesis’ (1910), Biddle (trans.), p. 77 (1997 English ed.).

[23] ‘This suggests that we are not dealing with a literary dependence or even a tradition dependence as much as we are dealing with two literary perspectives on a single actual event.‘, Walton, ‘Ancient Israelite Literature in Its Cultural Context: A Survey of Parallels’, p. 40 (1994).

[24] ‘The story may have arisen from a specific historical flood that took place in parts of southern Mesopotamia around 2900.’, Tigay, ‘The Evolution of the Gilgamesh Epic’, p. 214 (2002).

[25] ‘Could not stories be shared by the Bible and surrounding cultures because they are both based on a historical event? Both Scripture and Mesopotamian literature mention a flood because there indeed was a flood.’, Hamilton, ‘Handbook on the Pentateuch: Genesis, Exodus, Leviticus, Numbers, Deuteronomy’, p. 66(2005).

[26] ‘However, there are more options than simply concluding that the Bible borrowed from Babylon. An equally plausible explanation is that both traditions go back to a real event.’, Longman, ‘How to read Genesis’, pp. 86-87 (2005).

[27] ‘On the basis of substantial historical evidence, coupled with many parallel words and phrases, what reasonable conclusions could we make? Here are just three: 1. There is a likelihood that a flood event actually happened. Why would the Akkadians, Sumerians, and Hebrews invent such a story unless there was some historical basis? 2. considering the parallel accounts are describing a historical event in the region of southern Mesopotamia about 2900 B.C., then Genesis also is describing the same historical, regional flood, and not a global deluge. 3. A regional flood would have brought judgment to those in the region. Judgment would have been specific to the sinful Adamite population, those answerable to God, rather than a universal pronouncement upon all mankind everywhere.’, Fischer, ‘Historical Genesis: from Adam to Abraham’, p. 140 (2008).